You are on page 1of 80

1

THI V LI GII
CHN I TUYN QUC GIA
D THI OLYMPIC TON QUC T
CA VIT NAM
T NM 2005 N NM 2010
2




PHN I

*****
BI
3

THI CHN I TUYN QUC GIA
D THI IMO 2005

*Ngy thi th nht.
Bi 1. Cho tam gic ABC c (I) v (O) ln lt l cc ng trn ni tip, ngoi tip.
Gi D, E, F ln lt l tip im ca (I) trn cc cnh BC, CA, AB. Gi , ,
A B C
ln lt l
cc ng trn tip xc vi hai ng trn (I) v (O) ln lt ti cc im D, K (vi ng
trn
A
); ti E, M (vi ng trn
B
) v ti F, N (vi ng trn
C
). Chng minh rng:
1. Cc ng thng , , DK EM FN ng quy ti P.
2. Trc tm ca tam gic DEF nm trn on OP.

Bi 2. Trn mt vng trn c n chic gh c nh s t 1 n n. Ngi ta chn ra
k chic gh. Hai chic gh c chn gi l k nhau nu l hai chic gh c chn lin
tip. Hy tnh s cch chn ra k chic gh sao cho gia hai chic gh k nhau, khng c t
hn 3 chic gh khc.
Bi 3. Tm tt c cc hm s : f tha mn iu kin:
3 3 3 3 3 3
( ) ( ( )) ( ( )) ( ( )) f x y z f x f y f z + + = + +

*Ngy thi th hai.
Bi 4. Chng minh rng:
3 3 3
3 3 3
3
( ) ( ) ( ) 8
a b c
a b b c c a
+ +
+ + +

trong , , a b c l cc s thc dng.
Bi 5. Cho s nguyn t ( 3) p p > . Tnh:
a)
1
2 2
2
1
2
2
p
k
k k
S
p p

=
( (
=
( (

nu 1 (mod 4) p .
b)
1
2
2
1
p
k
k
S
p

=
(
=
(

nu 1 (mod8) p .
Bi 6. Mt s nguyn dng c gi l s kim cng 2005 nu trong biu din
thp phn ca n c 2005 s 9 ng cnh nhau lin tip. Dy ( ) , 1, 2, 3,...
n
a n = l dy tng
ngt cc s nguyn dng tha mn
n
a nC < (C l hng s thc dng no ).
Chng minh rng dy s ( ) , 1, 2, 3,...
n
a n = cha v hn s kim cng 2005.
www.VNMATH.com
4

THI CHN I TUYN QUC GIA
D THI IMO 2006

* Ngy thi th nht.
Bi 1. Cho tam gic ABC c H l trc tm. ng phn gic ngoi ca gc BHC ct cc
cnh AB, AC ln lt ti D v E. ng phn gic trong ca gc BAC ct ng trn ngoi
tip tam gic ADE ti im K. Chng minh rng ng thng HK i qua trung im ca BC.
Bi 2. Hy tm tt c cc cp s t nhin ( ) ; n k vi n l s nguyn khng m v k l
s nguyn ln hn 1 sao cho s :
2006 2 5
17 4.17 7.19
n n n
A = + + c th phn tch c thnh
tch ca k s nguyn dng lin tip.
Bi 3. Trong khng gian cho 2006 im m trong khng c 4 im no ng
phng. Ngi ta ni tt c cc im li bi cc on thng. S t nhin m gi l s tt nu
ta c th gn cho mi on thng trong cc on thng ni bi mt s t nhin khng
vt qu m sao cho mi tam gic to bi ba im bt k trong s cc im u c hai
cnh c gn bi hai s bng nhau v cnh cn li gn bi s ln hn hai s .
Tm s tt c gi tr nh nht.
* Ngy thi th hai .
Bi 4. Chng minh rng vi mi s thc , , [1; 2] x y z , ta lun c bt ng thc sau :
1 1 1
( )( ) 6( )
x y z
x y z
x y z y z z x x y
+ + + + + +
+ + +
.
Hi ng thc xy ra khi v ch khi no ?
Bi 5. Cho tam gic ABC l tam gic nhn, khng cn, ni tip trong ng trn tm
O bn knh R. Mt ng thng d thay i sao cho d lun vung gc vi OA v lun ct cc
tia AB, AC. Gi M, N ln lt l giao im ca ng thng d v cc tia AB, AC. Gi s cc
ng thng BN v CN ct nhau ti K; gi s ng thng AK ct ng thng BC.
1. Gi P l giao ca ng thng AK v ng thng BC. Chng minh rng ng trn
ngoi tip ca tam gic MNP lun i qua mt im c nh khi d thay i.
2. Gi H l trc tm ca tam gic AMN. t BC = a v l l khong cch t im A n HK.
Chng minh rng ng thng HK lun i qua trc tm ca tam gic ABC.
T suy ra:
2 2
4 l R a . ng thc xy ra khi v ch khi no?
Bi 6. Cho dy s thc ( )
n
a c xc nh bi:

0 1
1 1
1, ( )
2 3
n n
n
a a a
a
+
= = + vi mi n = 1, 2, 3,
Chng minh rng vi mi s nguyn n, s
2
3
3 1
n
n
A
a
=

l mt s chnh phng v n c t
nht n c nguyn t phn bit.
www.VNMATH.com
5

THI CHN I TUYN QUC GIA
D THI IMO 2007

*Ngy thi th nht.

Bi 1. Cho hai tp hp A,B l tp hp cc s nguyn dng tha mn A B n = =
(vi n l s nguyn dng) v c tng cc phn t bng nhau. Xt bng vung n n .
Chng minh rng ta c th in vo mi vung ca bng mt s nguyn khng m
tha mn ng thi cc iu kin:
i/ Tng ca cc phn t mi hng l cc phn t ca tp A.
ii/ Tng ca cc phn t mi ct l cc phn t ca tp B.
iii/ C t nht
2
( 1) n k + s 0 trong bng vi k l s cc phn t chung ca A v B.
Bi 2. Cho tam gic nhn ABC vi ng trn ni tip I. Gi ( )
a
k l ng trn c
tm nm trn ng cao ca gc A, i qua im A v tip xc trong vi ng trn (I) ti
1
A .
Cc im
1 1
, B C xc nh tng t .
1/ Chng minh
1 1 1
, , AA BB CC ng qui ti P.
2/ Gi ( ), ( ), ( )
a b c
J J J ln lt l cc ng trn i xng vi ng trn bng tip
cc gc A, B, C ca tam gic ABC qua trung im BC, CA, AB.
Chng minh P l tm ng phng ca 3 ng trn ni trn.
Bi 3. Cho tam gic ABC. Tm gi tr nh nht ca biu thc sau:
2 2 2 2 2 2
2 2 2
cos cos cos cos cos cos
2 2 2 2 2 2
cos cos cos
2 2 2
A B B C C A
S
C A B
= + + .
*Ngy thi th hai.

Bi 4. Tm tt c cc hm s lin tc : f tha mn:

2
1
( ) ( )
3 9
x
f x f x = + + vi mi x.
Bi 5. Cho A l tp con cha 2007 phn t ca tp:{1, 2, 3,..., 4013, 4014} tha mn
vi mi , a b A th a khng chia ht cho b. Gi mA l phn t nh nht ca A.
Tm gi tr nh nht ca mA vi A tha mn cc iu kin trn.
Bi 6. Cho a gic 9 cnh u (H). Xt ba tam gic vi cc nh l cc nh ca a gic
(H) cho sao cho khng c hai tam gic no c chung nh.
Chng minh rng c th chn c t mi tam gic 1 cnh sao cho 3 cnh ny bng nhau.
www.VNMATH.com
6

THI CHN I TUYN QUC GIA
D THI IMO 2008
*Ngy thi th nht.
Bi 1. Trong mt phng cho gc xOy. Gi M, N ln lt l hai im ln lt nm trn
cc tia Ox, Oy. Gi d l ng phn gic gc ngoi ca gc xOy v I l giao im ca trung
trc MN vi ng thng d. Gi P, Q l hai im phn bit nm trn ng thng d sao
cho IM IN IP IQ = = = , gi s K l giao im ca MQ v NP.
1. Chng minh rng K nm trn mt ng thng c nh.
2. Gi d1 l ng thng vung gc vi IM ti M v d2 l ng thng vung gc vi IN
ti N. Gi s cc ng thng d1, d2 ct ng thng d ti E, F. Chng minh rng cc ng
thng EN, FM v OK ng quy.
Bi 2. Hy xc nh tt c cc s nguyn dng m sao cho tn ti cc a thc vi h
s thc ( ), ( ), ( , ) P x Q x R x y tha mn iu kin:
Vi mi s thc a, b m
2
0
m
a b = , ta lun c ( ( , )) P R a b a = v ( ( , )) Q R a b b = .
Bi 3. Cho s nguyn n > 3. K hiu T l tp hp gm n s nguyn dng u tin.
Mt tp con S ca T c gi l tp khuyt trong T nu S c tnh cht: Tn ti s nguyn
dng c khng vt qu
2
n
sao cho vi
1 2
, s s l hai s bt k thuc S ta lun c
1 2
s s c .
Hi tp khuyt trong T c th c ti a bao nhiu phn t ?
*Ngy thi th hai.
Bi 4. Cho m, n l cc s nguyn dng. Chng minh rng (2 3) 1
n
m+ + chia ht cho
6m khi v ch khi 3 1
n
+ chia ht cho 4m.
Bi 5. Cho tam gic ABC nhn, khng cn c O l tm ng trn ngoi tip.
Gi AD, BE, CF l cc ng phn gic trong ca tam gic. Trn cc ng thng AD, BE, CF
ln lt ly cc im L, M, N sao cho
AL BM CN
k
AD BE CF
= = = (k l mt hng s dng).
Gi (O1), (O2), (O3) ln lt l cc ng trn i qua L, tip xc vi OA ti A ; i qua M, tip
xc vi OB ti B v i qua N, tip xc vi OC ti C.
1. Chng minh rng vi
1
2
k = , ba ng trn (O1), (O2), (O3) c ng hai im chung
v ng thng ni hai im chung i qua trng tm tam gic ABC.
2. Tm tt c cc gi tr k sao cho 3 ng trn (O1), (O2), (O3) c ng hai im chung.
Bi 6. K hiu M l tp hp gm 2008 s nguyn dng u tin. T tt c cc s
thuc M bi ba mu xanh, vng, sao cho mi s c t bi mt mu v mi mu u
c dng t t nht mt s. Xt cc tp hp sau:
3
1
{( , , ) , S x y z M = trong x, y, z c cng mu v ( ) 0 (mod 2008)} x y z + + ;
3
2
{( , , ) , S x y z M = trong x, y, z i mt khc mu v ( ) 0 (mod 2008)} x y z + + .
Chng minh rng
1 2
2 S S > . (K hiu
3
M l tch - cc M M M ) .
www.VNMATH.com
7

THI CHN I TUYN QUC GIA
D THI IMO 2009

*Ngy thi th nht.
Bi 1. Cho tam gic nhn ABC ni tip ng trn (O). Gi
1 1 1
, , A B C v
2 2 2
, , A B C
ln lt l cc chn ng cao ca tam gic ABC h t cc nh A, B, C v cc im i xng
vi
1 1 1
, , A B C qua trung im ca cc cnh , , BC CA AB. Gi
3 3 3
, , A B C ln lt l cc giao
im ca ng trn ngoi tip cc tam gic
2 2 2 2 2 2
, , AB C BC A CA B vi (O).
Chng minh rng:
1 3 1 3 1 3
, , A A B B C C ng quy.
Bi 2. Cho a thc
3 2
( ) 1 P x rx qx px = + + + trong , , p q r l cc s thc v 0 r > .
Xt dy s ( )
n
a xc nh nh sau:
2
1 2 3
3 2 1
1, ,
. . . , 0
n n n n
a a p a p q
a p a q a r a n
+ + +
= = =


Chng minh rng: nu a thc ( ) P x c mt nghim thc duy nht v khng c
nghim bi th dy s ( )
n
a c v s s m.
Bi 3. Cho cc s nguyn dng , a b sao cho , a b v ab u khng phi l s chnh
phng. Chng minh rng trong hai phng trnh sau:

2 2
2 2
1
1
ax by
ax by
=
=

c t nht mt phng trnh khng c nghim nguyn dng.
*Ngy thi th hai.
Bi 4. Tm tt c cc s thc r sao cho bt ng thc sau ng vi mi a, b, c dng:
3
1

2
a b c
r r r r
b c c a a b
| || || | | |
+ + + +
| | | |
+ + +
\ \ \ \

Bi 5. Cho ng trn (O) c ng knh AB v M l mt im bt k nm trong (O),
M khng nm trn AB. Gi N l giao im ca phn gic trong gc M ca tam gic AMB vi
ng trn (O). ng phn gic ngoi gc AMB ct cc ng thng NA, NB ln lt ti P,
Q. ng thng MA ct ng trn ng knh NQ ti R, ng thng MB ct ng trn
ng knh NP ti S v R, S khc M.
Chng minh rng: ng trung tuyn ng vi nh N ca tam gic NRS lun i qua
mt im c nh khi M di ng pha trong ng trn.
Bi 6. Mt hi ngh ton hc c tt c 6 4 n + nh ton hc phi hp vi nhau ng
2 1 n + ln ( ) 1 n . Mi ln hp, h ngi quanh mt ci bn 4 ch v n ci bn 6 ch, cc v
tr ngi chia u khp mi bn. Bit rng hai nh ton hc ngi cnh hoc i din nhau
mt cuc hp ny th s khng c ngi cnh hoc i din nhau mt cuc hp khc.
a/ Chng minh rng Ban t chc c th xp c ch ngi nu 1 n = .
b/ Hi rng Ban t chc c th sp xp c ch ngi c hay khng vi mi 1 n > ?
www.VNMATH.com
8

THI CHN I TUYN QUC GIA
D THI IMO 2010

* Ngy thi th nht.

Bi 1. Cho tam gic ABC khng vung ti A c ng trung tuyn AM. Gi D l mt
im di ng trn ng thng AM. Gi
1 2
( ), ( ) O O l cc ng trn i qua D, tip xc vi
BC ln lt ti B v C. Gi P, Q ln lt l giao im ca ng thng AB vi ng trn
1
( ) O , ng thng AC vi ng trn
2
( ) O . Chng minh rng:
1. Tip tuyn ti P ca
1
( ) O v tip tuyn ti Q ca
2
( ) O phi ct nhau ti mt im.
Gi giao im l S.
2. im S lun di chuyn trn mt ng thng c nh khi D di ng trn AM.
Bi 2. Vi mi s n nguyn dng, xt tp hp sau :
{ }
11( ) 10( ) | 1 , 10
k h
n
T k h n n k h = + + + .
Tm tt c gi tr ca n sao cho khng tn ti , ;
n
a b T a b sao cho ( ) a b chia ht cho 110.
Bi 3. Gi mt hnh ch nht c kch thc 1 2 l hnh ch nht n v mt hnh
ch nht c kch thc 2 3 , b i 2 gc cho nhau (tc l c 4 vung nh) l hnh ch
nht kp. Ngi ta ghp kht cc hnh ch nht n v hnh ch nht kp ny li vi nhau
c mt bng hnh ch nht c kch thc l 2008 2010 .
Tm s b nht cc hnh ch nht n c th dng ghp.

* Ngy thi th hai.
Bi 4. Cho , , a b c l cc s thc dng tha mn iu kin:
1 1 1
16( ) a b c
a b c
+ + + + .
Chng minh rng:

3 3 3
1 1 1 8
9 ( 2( )) ( 2( )) ( 2( )) a b a c b c b a c a c b
+ +
+ + + + + + + + +
.
Hi ng thc xy ra khi no?
Bi 5: Trong mt hi ngh c n nc tham gia, mi nc c k i din ( ) 1 n k > > .
Ngi ta chia . n k ngi ny thnh n nhm, mi nhm c k ngi sao cho khng c hai
ngi no cng nhm n t cng mt nc.
Chng minh rng c th chn ra mt nhm gm n ngi sao cho h thuc cc nhm khc
nhau v n t cc nc khc nhau.
Bi 6: Gi
n
S l tng bnh phng cc h s trong khai trin ca nh thc (1 )
n
x + ,
trong n l s nguyn dng; x l s thc bt k.
Chng minh rng:
2
1
n
S + khng chia ht cho 3 vi mi n.
www.VNMATH.com
9





PHN II
*****
LI GII
10

LI GII THI
CHN I TUYN QUC GIA D THI IMO 2005

Bi 1. Cho tam gic ABC c (I) v (O) ln lt l cc ng trn ni tip, ngoi tip.
Gi D, E, F ln lt l tip im ca (I) trn cc cnh BC, CA, AB. Gi , ,
A B C
ln lt
l cc ng trn tip xc vi hai ng trn (I) v (O) ln lt ti cc im D, K (vi ng
trn
A
); ti E, M (vi ng trn
B
) v ti F, N (vi ng trn
C
). Chng minh rng:
1. Cc ng thng DK, EM, FN ng quy ti P.
2. Trc tm ca tam gic DEF nm trn on OP.
1. Trc ht, ta s chng minh b sau:
Cho ba ng trn (O
1
), (O
2
), (O
3
) c bn
knh i mt khc nhau; A, B, C ln lt l tm v
t ca cc cp ng trn (O
1
) v (O
2
), (O
2
) v
(O
3
), (O
3
) v (O
1
).
Chng minh rng nu trong cc tm v t , c
ba tm v t ngoi hoc hai tm v t trong, mt
tm v t ngoi th A, B, C thng hng.
*Chng minh:
Gi
1 2 3
, , R R R ln lt l bn knh ca cc ng
trn
1 2 3
( ), ( ), ( ) O O O , cc gi tr
1 2 3
, , R R R ny i
mt khc nhau.
Theo tnh cht v tm v t, ta c:
1 1
2 2
( 1)
a
AO R
R AO
= .
Tng t:
2 2
3 3
( 1)
b
BO R
R BO
= ,
3 3
1 1
( 1)
c
CO R
R CO
= , trong
, mi s , , a b c nhn gi tr l 0 (khi n l tm v t ngoi) hoc 1 (khi n l tm v t trong).
Theo gi thit trong a, b, c c ba gi tr l 0 hoc hai gi tr 0, mt gi tr 1. T :
3 1 2
2 3 1
. . 1
CO AO BO
AO BO CO
= , theo nh l Menelaus o cho tam gic
1 2 3
OO O , ta c: A, B, C thng hng.
B c chng minh.
*Tr li bi ton:
Gi P l tm v t trong ca hai ng trn (O) v (I). D thy: D l im tip xc ngoi
ca
A
v (I) nn cng chnh l tm v t trong ca hai ng trn ny; K l im tip xc trong
ca hai ng trn
A
v (O) nn l tm v t ngoi ca hai ng trn ny. Theo b trn th
', , P D K thng hng hay ng thng DK i qua P. Tng t, cc ng thng EM v FN cng
i qua P; tc l ba ng thng DK, EM, FN ng quy v im P chnh l im P ca bi.
B
C
A
O
1
O
2
O
3
www.VNMATH.com
11

2. Ta chng minh b sau:
Cho tam gic ABC c (O), (I) ln lt l tm ng trn ngoi tip, ni tip ca tam gic
ABC. ng trn (I) tip xc vi cc cnh BC, CA, AB ln lt ti D, E, F. Chng minh rng
trc tm H ca tam gic DEF nm trn ng thng OI.

* Chng minh:
Gi M, N, P ln lt l trung im ca cc
on EF, FD, DE. D thy AI l trung trc ca
on EF nn M thuc ng thng AI hay A, M, I
thng hng. Tng t: B, N, I v C, P, I cng
thng hng. Xt php nghch o tm I, phng
tch
2
r vi r l bn knh ng trn (I).
D thy: tam gic IEA vung ti E c EM l
ng cao nn:
2 2
. IM IA IE r = = , suy ra:
: M A . Tng t: : , N B P C .
Do : : MNP ABC . Gi E l tm
ng trn ngoi tip ca tam gic MNP th
: E O , suy ra: E, I, O thng hng.

Hn na, I l tm ng trn ngoi tip tam gic DEF, E l tm ng trn ngoi tip tam
gic MNP cng chnh l tm ng trn Euler ca tam gic DEF ny nn E, I, H thng hng.
T suy ra H, I, O thng hng. B c chng minh.

* Tr li bi ton:
Gi H l trc tm tam gic DEF th theo b trn:
H, I, O thng hng.
Theo cu 1/, im P nm trn on OI.
Suy ra: 4 im H, I, P, O thng hng.
T suy ra trc tm H ca tam gic DEF nm trn
ng thng OI.
Ta c pcm.
N
M
H
K
P
F
E
D
I
O
B
C
A
H
P
N
M
F
E
D
I
O
A
B C
www.VNMATH.com
12

Bi 2. Trn mt vng trn c n chic gh c nh s t 1 n n. Ngi ta chn ra k
chic gh. Hai chic gh c chn gi l k nhau nu l hai chic gh c chn lin tip.
Hy tnh s cch chn ra k chic gh sao cho gia hai chic gh k nhau, khng c t hn 3
chic gh khc.

*Trc ht, ta chng minh b sau:
Cho n im phn bit nm trn ng thng c t mt trong hai mu, xanh hoc
tha mn cc iu kin sau:
- C ng k im c t mu xanh.
- Gia hai im mu xanh lin tip c t nht p im c t mu (tnh t tri sang).
- bn phi im mu xanh cui cng c t nht p im c t mu .
Khi , s cch t mu l:
k
n kp
C

.
*Chng minh: nh s cc im cho l 1, 2, 3,..., n . t tng ng mi cch t mu
vi mt b k cc s nguyn dng
1 2
( , ,..., )
k
i i i trong
1 2
, ,...,
k
i i i l cc im c t mu xanh.
D thy tng ng ni trn chnh l mt song nh t tp cc cch t mu n tp hp T sau:
1 2 1
{( , ,..., ) | {1, 2,..., }, 1, ; , 1, 1}
k s s s
T i i i i n p s k i i p i k
+
= = > = .
Xt nh x sau:

1 2 1
' {( , ,..., ) | {1, 2,..., }, , 1, }
k t t t
T T j j j j n kp j j t k
+
= > = .
Ta s chng minh nh x ny l mt song nh.
Tht vy:
*Xt mt b
1 2
( , ,..., ) '
k
j j j T . Khi , ta xt tip b:
1 2 3
( , , 2 ,..., ( 1) )
k
j j p j p j k p + + + .
Do 1
t
j n kp nn phn t ln nht ca b ny l ( 1)
k
j k p + c gi tr khng vt qu
( 1) n kp k p n p + = . T suy ra: ( 1) {1, 2,..., }, 1
t
j t p n kp t + .
Hn na: | | | |
1 1
( 1) ( )
t t t t
j tp j t p j j p p
+ +
+ + = + > .
T suy ra b
1 2 3
( , , 2 ,..., ( 1) )
k
j j p j p j k p T + + + .
Do , tng ng ny l mt ton nh.
*Xt b
1 2
( , ,..., )
k
i i i T . Khi , hon ton tng t trn, ta cng chng minh c b
1 2 2
( , , 2 ,..., ( 1) ) '
k
i i p i p i k p T .
Ta s chng minh rng nu c hai b khc nhau
1 2 1 2
( , ,..., ), ( ' , ' ,..., ' )
k k
i i i i i i T th cc b tng
ng thuc T= ' T ca chng cng phi khc nhau. Nhng iu ny l hin nhin do hai b ny l
khc nhau nn tn ti ch s s sao cho '
s s
i i , khi ( 1) ' ( 1)
s s
i s p i s p .
Suy ra, tng ng ny cng l mt n nh.
Vy tng ng ' T T l mt song nh.
Nhn xt trn c chng minh.
Do : | | | ' |
k
n kp
T T C

= = . B c chng minh.
www.VNMATH.com
13

*Tr li bi ton:
Ta xt tng qut gi tr 3 trong bi bi gi tr p tng ng vi b trn.
nh s cc gh trong bi theo chiu kim ng h l
1 2
, ,...,
n
A A A (xem nh l cc im nm
trn mt vng trn) ; mi gh c chn xem nh c t mu xanh v khng c chn xem
nh c t mu ; gi X l tp hp tt c cc cch t mu k im trong n im cho tha
mn bi.
Xt phn hoch: ' '' ' " X X X X X X = = + .
trong ' X l cch t mu tha mn c mt im c t mu xanh thuc
1 2 3
{ , , ,..., }
p
A A A A v
'' \ ' X X X = , khi r rng, vi mi phn t thuc " X th khng c im no c t mu
xanh thuc
1 2 3
{ , , ,..., }
p
A A A A , tc l mi im trong tp ny u c t mu . Ta ct ng
trn ngay ti im
1
,
p p
A A
+
th r rng s to c mt ng thng tha mn tt c iu kin
ca b nu trn, suy ra: ''
k
n kp
X C

= . Ta ch cn cn tnh s phn t ca ' X .
Xt tp hp '
i
X trong mi phn t ca '
i
X c ng mt im
i
A c t mu xanh,
1, i p = ; khi r rng ' ' ,
i j
X X i j = v
1
' '
p
i
i
X X
=
=

.
Vi mi 1, i p = , theo b trn, ta thy:
1 1
1 ( 1) 1
k k
n p k p n kp
C C


= , tc l cc tp '
i
X ny c cng s phn t. Suy ra:
1
1
'
k
n kp
X pC


= .
Do :
1
1
' ''
k k
n kp n kp
X X X C pC


= + = + .
Thay 3 p = , ta c s cch chn gh tng ng trong bi l
1
3 3 1
3
k k
n k n k
C C


+ .
Vy s cch chn gh tha mn tt c cc iu kin ca bi l:
1
3 3 1
3
k k
n k n k
C C


+ .
www.VNMATH.com
14

Bi 3. Tm tt c cc hm s : f tha mn iu kin:
3 3 3 3 3 3
( ) ( ( )) ( ( )) ( ( )) f x y z f x f y f z + + = + +

* Trc ht, ta s chng minh b sau:
Vi mi s nguyn dng ln hn 10, lp phng ca n u c th biu din c di
dng tng ca 5 lp phng ca cc s nguyn khc c gi tr tuyt i nh hn n.

* Tht vy:
Ta cn tm mi lin h vi s 10 n > trong tng trng hp n chn v n l.
- Vi n l s l, t 2 1 n k = + .
Ta cn tm mt ng thc ng vi mi k m trong 2 1 k + l biu thc c gi tr tuyt
i ln nht, cc biu thc cn li phi l nh thc bc nht c h s ca k ln nht l 2. Khi
kh
3
8k xut hin trong ( )
3
2 1 k + , ta chn ( )
3
2 1 k ; ta thy vn cn s hng cha k bc
hai trong , ta chn tip hai biu thc khc c cha k cng hai hng s bng cch dng tham s
nh sau:
Gi s hai biu thc cn tm c dng ( ), ( ); , ak b ak b a b + v hai s cn tm l
, c d , tc l:
( ) ( ) ( )
3 3 3 3 3 3
2 2 2 3 3 3
2 2 3 3 3
(2 1) (2 1) ( ) ( )
24 2 6 2
(24 6 ) (2 2 ) 0
k k ak b ak b c d
k a bk b c d
k a b b c d
( ( ( + = + + +

+ = + + +
+ =

Ta cn chn , , , a b c d sao cho
2 3 3 3
4, 2 2 a b b c d = + + = trong 2 a .
D thy 2 a v nu 2 a = th t
2
4 1 a b b = = , trng vi biu thc cn nh gi; do ,
1, 4 a b = = , suy ra:
3 3
126 c d + = , ta chn c 5, 1 c d = = .
Do :

3 3 3 3 3 3
(2 1) (2 1) ( 4) (4 ) ( 5) ( 1) k k k k + = + + + + + (1)
Th li, ta thy biu thc ny ng vi mi k.
- Vi n l s chn, t 2 2 n k = + .
Lp lun hon ton tng t, ta c c ng thc sau:
3 3 3 3 3 3
(2 2) (2 2) ( 8) (8 ) ( 10) ( 2) k k k k + = + + + + + (2)
B c chng minh.

*Tr li bi ton:
Trong ng thc , thay 0 x y z = = = , ta c:
3 2
(0) 3 (0) (0) 0 3 (0) 1 f f f f = = = .
Do hm ny ch ly gi tr trn : f nn khng th c
2
3 (0) 1 f = , tc l (0) 0 f = .
www.VNMATH.com
15

- Thay 0 y z = = , ta c:
3 3 3 3 3
( ) ( ( )) ( (0)) ( (0)) ( ( )) f x f x f f f x = + + = .
- Li thay y z = , ta c:
3 3 3 3 3 3
( ) ( ( )) ( ( )) ( ( )) ( ( )) ( ( )) 0 ( ) ( ), f x f x f y f y f y f y f y f y y = + + + = = .
Ta s chng minh rng:
3
: ( ) . (1) k f k k f = bng quy np. (*)
*Tht vy:
- Vi 1 k = , trong gi thit, thay 1, 0 x y z = = = , ta c
3 3
(1) (1) ( 1) (1) f f f f = =
- Vi 2 k = , trong gi thit, thay 1, 0 x y z = = = , ta c
3 3
(2) 2 (1) ( 2) 2 (1) f f f f = =
- Vi 3 k = , trong gi thit, thay 1 x y z = = = , ta c
3 3
(3) 3 (1) ( 3) 3 (1) f f f f = =
- Thay 2, 0 x y z = = = , ta c
3 3 3 3
(8) (2) (2 (1)) 8 (1) ( 8) 8 (1) f f f f f f = = = = .
- Thay 2, 1, 0 x y z = = = , ta c
3 3 3 3
(9) (2) (1) 9 (1) ( 9) 9 (1) f f f f f f = + = = .
- Thay 2, 1 x y z = = = , ta c:
3 3 3 3
(10) (2) 2 (1) 10 (1) ( 10) 10 (1) f f f f f f = + = = .
- Thay 2, 1, 0 x y z = = = , ta c:
3 3 3 3
(7) (2) (1) 7 (1) ( 7) 7 (1) f f f f f f = = = .
- Thay 2, 1 x y z = = = , ta c:
3 3 3 3
(6) (2) 2 (1) 6 (1) ( 6) 6 (1) f f f f f f = = = .
- Trong ng thc (1) ca b trn, ta thay 2 k = , suy ra:
3 3 3 3 3 3
5 3 6 2 ( 5) ( 1) = + + + + hay
3 3 3 3 3 3
(5 5 1 ) (3 6 2 ) f f + + = + + .

3 3 3 3 3 3 3
2 (5) (1 ) (3) (6) (2) (5) 5 (1) ( 5) 5 (1) f f f f f f f f f + = + + = = .
- Trong ng thc (2) ca b trn, ta thay 1 k = , suy ra:
3 3 3 3 3 3
4 0 9 7 ( 10) ( 2) = + + + + hay
3 3 3 3 3 3
(4 10 2 ) (9 7 0 ) f f + + = + + .
3 3 3 3 3 3 3 3
(4) (10) (2) (9) (7) (0) (4) 4 (1) ( 4) 4 (1) f f f f f f f f f f + + = + + = = .
Nh th, ta chng minh c (*) ng vi mi 10 k .
Vi 10 k > , xt 0 k > th theo b trn, lp phng ca k u c th biu din c
di dng tng ca 5 lp phng khc c gi tr tuyt i nh hn n.
Hn na, d thy rng vi , , , , , a b c d e f tha
3 3 3 3 3 3
a b c m n p + + = + + v ta c:
3 3 3 3 3
( ) (1), ( ) (1), ( ) (1), ( ) (1), ( ) (1) f b bf f c cf f m mf f n nf f p pf = = = = = th
3
( ) (1) f a af = .
T , suy ra
3
( ) (1), 10 f k kf k = > .
Vi 10 k < th
3 3
( ) ( ) ( (1)) (1) f k f k kf kf = = = .
Do , theo nguyn l quy np (*) c chng minh.
Mt khc, trong gi thit cho, thay 1, 0 x y z = = = , ta c:
3
(1) (1) (1) 1 (1) 0 f f f f = = = .
- Nu (1) 1 f = th ( ) , f k k k = , th li thy tha.
- Nu (1) 0 f = th ( ) 0, f k k = , th li thy tha.
- Nu (1) 1 f = th ( ) , f k k k = , th li thy tha.
Vy tt c hm s cn tm l ( ) , f k k k = ; ( ) , f k k k = v ( ) 0, f k k = .

www.VNMATH.com
16

Bi 4. Chng minh rng:
3 3 3
3 3 3
3
( ) ( ) ( ) 8
a b c
a b b c c a
+ +
+ + +

trong , , a b c l cc s thc dng.

*Trc ht, ta s chng minh b sau:
Nu , , , a b c d l cc s thc dng c tch bng 1 th:
2 2 2 2
1 1 1 1
1
(1 ) (1 ) (1 ) (1 ) a b c d
+ + +
+ + + +
.
Tht vy:
Ta thy vi hai s thc dng ty th:
2 2
1 1 2
(1 ) (1 ) 1 x y xy
+
+ + +
(*)
2 2
2 2 2
2
2 2 2
2 2 3 3 2 2
2 2 2 2 2 2
( 1) ( 1) 1
(*) ( 1) ( 1) (1 ) ( 1)
( 1) 1
( 2 2 2)(1 ) ( ) 2( ) 1
( 2 2 2) ( 2 2 2 )
( 2 2 2 ) (2 2 2 ) 1
1
x y
x y xy xy x y
xy x y xy
x y x y xy xy x y xy x y
x y x y x y xy x y xy xy
x y x y x y xy xy xy x y
x
+ + +
( + + + + + + +

+ + + +
+ + + + + + + + + + +
+ + + + + + + + +
+ + + + + + + + +
+
2 2 2 2 2 2
( ) 2 ( ) (1 ) 0 y x y xy x y xy x y xy + + +

Do :
2 2 2 2
1 1 1 1 1 1 2 2
1
(1 ) (1 ) (1 ) (1 ) 1 1 1 2
ab cd ab cd
a b c d ab cd ab cd abcd ab cd
+ + + +
+ + + + = = =
+ + + + + + + + + + +
.
Do b c chng minh.
ng thc xy ra khi v ch khi 1 a b c d = = = = .
Trong b trn, thay , , , 1 a x b y c z d = = = = , ta c kt qu sau:
Vi x, y, z l cc s thc dng v 1 xyz = th:
2 2 2
1 1 1 3
(1 ) (1 ) (1 ) 4 x y z
+ +
+ + +
. ng thc xy ra khi v ch khi 1 x y z = = = .

*Tr li bi ton cho:
t , , , , 0; 1
b c a
x y z x y z xyz
a b c
= = = > = .
BT cho ban u tng ng vi:
3 3 3
3 3 3
1 1 1 3 1 1 1 3
8 (1 ) (1 ) (1 ) 8
(1 ) (1 ) (1 )
b c a
x y z
a b c
+ + + +
+ + +
+ + +
.
Theo BT Cauchy cho cc s dng, ta c:
www.VNMATH.com
17

3
3 3 6 2 3 2
1 1 1 1 3 1 1 3 1 1
3 . .
(1 ) (1 ) 8 8.(1 ) 2 (1 ) (1 ) 4 (1 ) 16 x x x x x x
+ + =
+ + + + + +
.
Hon ton tng t:
3 2
1 3 1 1
.
(1 ) 4 (1 ) 16 y y

+ +
,
3 2
1 3 1 1
.
(1 ) 4 (1 ) 16 z z

+ +
.
Cng tng v cc BT ny li, ta c:
3 3 3 2 2 2
1 1 1 3 1 1 1 3
.
(1 ) (1 ) (1 ) 4 (1 ) (1 ) (1 ) 16 x y z x x x
(
+ + + +
(
+ + + + + +

.
Ta cn chng minh:
2 2 2
3 1 1 1 3 3
.
4 (1 ) (1 ) (1 ) 16 8 x x x
(
+ +
(
+ + +

2 2 2
1 1 1 3
(1 ) (1 ) (1 ) 4 x x x
+ +
+ + +

vi x, y, z tha mn cc iu kin nu. (**)
Theo b trn th (**) ng.
Vy ta c pcm.
Du ng thc xy ra khi v ch khi 1 x y z a b c = = = = = .

www.VNMATH.com
18

Bi 5. Cho s nguyn t ( 3) p p > . Tnh:
a.
1
2 2
2
1
2
2
p
k
k k
S
p p

=
| | ( (
=
|
( (
\

nu 1 (mod 4) p .
b.
1
2
2
1
p
k
k
P
p

=
(
=
(

nu 1 (mod8) p .

*Trc ht, ta s chng minh hai b sau:
(1) B 1: Vi p l s nguyn t tha 1 (mod 4) p th mi s t nhin a vi:
1
1
2
p
a


s tn ti duy nht s t nhin b tha
1
1
2
p
b p
+
v:
2 2
0(mod ) a b p + .
*Chng minh: Theo nh l Wilson: ( 1)! 1(mod ) p p .
Vi mi
1
1, 2, 3,...,
2
p
k

= , ta thy:
2
(mod ) ( ) (mod ) p k k p k p k k p .
Kt hp vi gi thit
1
1(mod 4) 2
2
p
p

, ta c:
2 2
1
2
1 1
1 ( 1)! ( 1) . ! ! (mod )
2 2
p
p p
p p

| | | | | | | |

| | | |
\ \ \ \
. t
2
1
! 1(mod )
2
p
p
| |
=
|
\
.
Vi mi
1
1
2
p
a

, ta chn
1
1
2
p
b p
+
tha
2 2 2
. (mod ) b a p , d thy b tn ti v duy
nht. Khi :
2 2 2 2
(1 ) 0(mod ) a b a p + + . B c chng minh.
(2) B 2:
Vi x l s thc bt k th | | | | 2 2 x x bng 1 nu
1
{ } 1
2
x < v bng 0 nu
1
0 { }
2
x < .
*Chng minh: Ta c: | | { } x x x = + . Suy ra:
| | | | | | | | | | | | | | 2 2 2[ ] 2{ } 2 [ ] { } 2{ } 2 { } 2{ } x x x x x x x x x = + + = = . Do :
-Nu
1
{ } 1
2
x < th | | | | | | 1 2{ } 2 2{ } 1 2 2 1 x x x x < = = .
- Nu
1
0 { }
2
x < th | | | | | | 0 2{ } 1 2{ } 0 2 2 0 x x x x < = = .
B c chng minh. *Tr li bi ton:
www.VNMATH.com
19

1. Ta thy tng cho l:
1
2 2
2
1
2
2
p
k
k k
S
p p

=
| | ( (
=
|
( (
\

c ng
1
2
p
s hng.
Theo b 2 th mi s hng trong tng nhn hai gi tr l 0 hoc 1. (1)
Theo b 1 th vi mi s t nhin a tha
1
1
2
p
a

th tn ti duy nht s t nhin b tha
1
1
2
p
b p
+
sao cho
2 2 2 2
0(mod ) ( ) 0(mod ) a b p a p b p + + ; do , tn ti duy nht
s t nhin ' a tha
1
1 '
2
p
a

sao cho
2 2
' 0(mod ) a a p + .
Gi , x y ln lt l s cc s d ca php chia
2
k cho p (
1
1
2
p
k

) c gi tr ln hn
1
2
p

v nh hn
1
2
p
. Theo nhn xt trn th x y = , hn na
1 1
2 4
p p
x y x y

+ = = = . (2)
T (1) v (2), ta c:
1
.1 .0
4
p
S x y

= + = .
Do , tng cn tm l
1
4
p
.
2. Do 1 (mod8) p nn tn ti a sao cho
2
2(mod ) a p .
(ta cng thy rng 1 (mod8) 1 (mod 4) p p ).
Ta c:
1 1 1 1
2 2 2 2 2 2 2
2 2 2 2
1 1 1 1
2 2 2
2
p p p p
k k k k
k k k k k k k
P S
p p p p p p p

= = = =
| | | | | | ( ( ( ( ( ( (
= = =
| | |
( ( ( ( ( ( (
\ \ \

.
Ta cn tnh:
1 1 1 1 1
2 2 2 2 2 2 2 2 2
2 2 2 2 2
1 1 1 1 1
2 2 2 2
p p p p p
k k k k k
k k k k k k k k k
p p p p p p p p p

= = = = =
| | | | | | ( ( | | | |
= =
| | | ` ` ` ` | | ( (
\ ) ) \ ) ) \ \ \

,
trong 1 (mod8) p .
Theo nhn xt trn th tp hp cc s d khi chia
2
1
,1
2
p
k k

cho p trng vi tp hp cc s
d khi chia
2
1
2 ,1
2
p
k k

cho p, tc l:
1
2 2
2
1
2
0
p
k
k k
p p

=
| |
=
| ` `
) ) \

, suy ra:
1 1
2 2 2 2
2 2
1 1
2 1
24
p p
k k
k k k p
p p p

= =
| | ( ( | |
= =
| | ( (
\ \

.
Vy
2
1 1 ( 1)( 5)
24 4 24
p p p p
P

= = .
www.VNMATH.com
20

Bi 6. Mt s nguyn dng c gi l s kim cng 2005 nu trong biu din
thp phn ca n c 2005 s 9 ng cnh nhau lin tip. Dy ( ) , 1, 2, 3,...
n
a n = l dy tng
ngt cc s nguyn dng tha mn
n
a nC < (C l hng s thc dng no ).
Chng minh rng dy s ( ), 1, 2, 3,...
n
a n = cha v hn s kim cng 2005.

Trc ht, ta s chng minh cc b sau:
(1)
1
1
lim
n
i
n
=
= +

.
(2) Nu trong h c s m( , 1) m m > : dy s ( )
n
a tng v trong dy khng c s
hng no c cha ch s 1 m th tng sau
1
1
n
i
i
a
=

hi t khi n tin ti v cc.


*Chng minh b (1):
Ta cn chng minh BT: ln( 1), 0 x x x > + > . Tht vy:
Xt hm s:
1
( ) ln( 1), 0 ( ) 1 0, 0
1 1
x
f x x x x f x x
x x
= + > = = > >
+ +
.
Do , hm s ( ) f x ng bin trn (0; ) + . Suy ra: ( ) (0) 0 ln( 1), 0 f x f x x x > = > + > .
Trong BT ny, thay x bi
1
0
x
> , ta cng c:
1 1 1 1 1
ln( 1) ln( ) ln( 1) ln , 0
x
x x x
x x x x x
+
> + > > + > . p dng vo tng cn chng minh:
| |
1 1
1
ln( 1) ln( ) ln( 1) ln1 ln( 1)
n n
i i
n n n n
n
= =
> + = + = +

, m | | lim ln( 1) n + = + nn:
1
1
lim
n
i
n
=
= +

. B c chng minh.
*Chng minh b (2):
t
1
k
s
n
=

l tng cc s t nhin c cha k ch s vit trong h c s m v khng c


cha ch s 1 m no.
Gi s mt s hng c k ch s no c dng:
1 2 1
...
k k
bb b b

, ch s th 1 phi khc 0 v
khc 1 m nn c 2 m cch chn, cc ch s cn li phi khc 1 m nn c 1 m cch chn.
Do , c ng
1
( 2).( 1)
k
m m

s c k ch s m trong biu din trong h c s m khng c
cha ch s 1 m , m mi s trong u ln hn
1 k
m

nn tng nghch o tng ng ca
chng s b hn
1
1
( 2).( 1)
k
k
m m
m


.
www.VNMATH.com
21

Hn na:
1
k
s
n
=

l tng cc s hng c cha k ch s trong h s m v khng c cha


ch s 1 m no nn n khng vt qu tng ca tt c cc s t nhin c cng dng m ta
va nh gi c, suy ra:
1
1
( 2).( 1)
k
k k
m m
s
m


< .
Do :
1
1
1
1 1 1 1
1 ( 2).( 1) 1 2
lim lim lim lim ( 2).( ) ( 2)
1
1
k n n n n
k
k k
i k k k
i
m m m m
s m m m
m
a m m
m

= = = =

= < = = =


.
Tc l tng ny hi t khi n tin ti v cc. B (2) c chng minh.

*Tr li bi ton:
t
2005
10 1 m m = l s t nhin c cha ng 2005 s 9 lin tip khi vit trong h
thp phn.
Ta cn chng minh trong dy cho, c v s s hng cha ch s 1 m .
Gi s trong dy ny khng c cha s hng no c ch s 1 m . Khi , theo b (2)
trn:
1
1
lim
n
i
i
a
=

l hu hn.
Hn na, theo gi thit: ,
n
a nC n < nn
1 1 1
1 1 1 1
lim lim .lim
n n n
i i i
i
a nC C n
= = =
> =

. Theo b
(1), gii hn ny tin ti v cc. Hai iu ny mu thun vi nhau chng t iu gi s trn
l sai, tc l dy cho c t nht mt s hng cha ch s 1 m , gi s l:
0
n
a .
Ta li xt dy con ca dy ban u:
0 0 0
1 2 3
, , ,...
n n n
a a a
+ + +

Dy ny c y tnh cht ca dy cho nn cng cha t nht mt s hng c cha ch s
1 m khc vi s
0
n
a trn (do y l dy tng).
Lp lun tng t nh th, dy con ny c thm mt s hng c cha ch s 1 m .
T suy ra dy cho c v s s hng cha ch s 1 m .
Vy dy s ( ) , 1, 2, 3,...
n
a n = cha v hn s kim cng 2005. y chnh l pcm.




www.VNMATH.com
22

LI GII THI CHN I TUYN QUC GIA
D THI IMO 2006

Bi 1. Cho tam gic ABC c H l trc tm. ng phn gic ngoi ca gc BHC ct
cc cnh AB, AC ln lt ti D v E. ng phn gic trong ca gc BAC ct ng trn
ngoi tip tam gic ADE ti im K.
Chng minh rng ng thng HK i qua trung im ca on BC.

Trc ht ta s chng minh ADE cn ti A.
Tht vy: V HD l phn gic gc ngoi ca

BHC nn:

0
1 1 1
( ) (90 ) (90 )
2 2 2
DHB HBC HCB ABC ACB BAC
(
= + = + =

.
Do :

0 0
1 1
90 90
2 2
ADE DBH DHB BAC BAC BAC = + = + = .
Tng t, ta cng c:

0
1
90
2
AED BAC = , suy ra:

ADE AED = , tc l tam gic ADE cn ti A.
Mt khc AK l phn gic

DAE
nn cng l trung trc ca on DE, do
AK chnh l ng knh ca ng trn
ngoi tip ADE .
T , ta c: KD AB , tng t ta cng
c:
KE AC
.
Gi P l giao im ca KD v HB, Q l
giao im ca KE v HC.
Ta c: , KP AB QH AB KP // QH.
Tng t, ta cng c: KQ // PH. Suy ra:
KPHQ l hnh bnh hnh, tc l HK i qua
trung im ca PQ.
Gi BB, CC l cc ng cao ca
tam gic ABC. Theo nh l Thals: DP // HC
'
PB DB
PH DC
= , QE // HB
'
QC EC
QH EB
= .
Theo tnh cht ng phn gic: ,
' ' ' '
DB HB EC HC
DC HC EB HB
= = .
V B, C, B, C cng thuc ng trn ng knh BC nn theo tnh cht phng tch:
. ' . '
' '
HB HC
HB HB HC HC
HC HB
= = . T cc iu ny, ta c:
PB QC
PH QH
= PQ // BC.
V HK i qua trung im ca PQ nn cng i qua trung im ca BC. Ta c pcm.
C'
B'
P
Q
K
D
E
H
A
B
C
www.VNMATH.com
23

Bi 2. Hy tm tt c cc cp s t nhin (n ; k) vi n l s nguyn khng m v k l s
nguyn ln hn 1 sao cho s :
2006 2 5
17 4.17 7.19
n n n
A = + + c th phn tch c thnh tch
ca k s nguyn dng lin tip.

Trc ht ta thy rng tch ca 4 s t nhin lin tip phi chia ht cho 8 v trong 4 s
c 1 s chia ht cho 4 v mt s chia 4 d 2.
T
2006 2 5
17 4.17 7.19
n n n
A = + + , suy ra :
- Nu n l s chn, ta c :
2006 2 5 2 10
17 1 (mod8), 4.17 4.1 (mod8), 7.19 7.3 7.3 7 (mod8)
n n n n

Suy ra : 12 4(mod8) A , tc l A khng chia ht cho 8.
- Nu n l s l, cng tng t :
2006 2 2 5
17 1 (mod8), 4.17 4.1 (mod8), 7.19 7.3 7.3 5(mod8)
n n n

Suy ra : 10 2(mod8) A , tc l A cng khng chia ht cho 8.
Tc l trong mi trng hp lun c A khng chia ht cho 8.
Suy ra nu k tha mn bi th 4 {2, 3} k k < .
Xt tng trng hp :
- Nu 2 k = : tn ti x t nhin sao cho ( 1) A x x = + .
+ Nu n = 0 th A = 12, x = 3, tha mn bi.
+ Nu n > 0 th r rng
1003 2 5
17 4.17 7.19
n n n
> + . Ta thy :
2006 2 5 2006
( 1) 17 4.17 7.19 17
n n n n
A x x = + = + + > , suy ra
1003
17
n
x > nhng
2006 1003
( 1) 17 17
n n
x x A + > + > , mu thun.
Do , trong trng hp ny khng c n tha mn bi.
- Nu 3 k = : tn ti x t nhin sao cho ( 1)( 1), 1 A x x x x = + ; d thy x phi l s chn (v
nu ngc li th A chia ht cho 8, mu thun). Ta thy :
12.( 1) 2.( 1) (mod5)
n n
A trong khi
2
( 1)( 1) ( 1) 0, 1(mod5) x x x x x + = , mu thun.
Do , trong trng hp ny khng c n tha mn bi.
Vy tt c cc cp s tha mn bi l ( ; ) (0; 2) n k = .
www.VNMATH.com
24

Bi 3. Trong khng gian cho 2006 im m trong khng c 4 im no ng phng. Ngi
ta ni tt c cc im li bi cc on thng. S t nhin m gi l s tt nu ta c th gn
cho mi on thng trong cc on thng ni bi mt s t nhin khng vt qu m sao
cho mi tam gic to bi ba im bt k trong s cc im u c hai cnh c gn bi
hai s bng nhau v cnh cn li gn bi s ln hn hai s .
Tm s tt c gi tr nh nht.

Do trong cc im cho khng c bn im no ng phng nn ba im bt k trong
chung lun to thnh mt tam gic. Gi S(n) l gi tr nh nht ca s tt ng vi n im trong
khng gian (n l s t nhin), ta s xc nh gi tr ca S(2006). Ta ch xt cc gi tr 4 n .
- Vi n = 4 th th trc tip, ta thy S(4) = 2. Bi v S(4) = 1 khng
tha mn nn (4) 2 S , ta s ch ra rng S(4) = 2 tha mn. C th
ta c th gn cc on thng nh sau : gn 4 on bt k bi s 1 v
2 on cn li bi s 2, r rng cc tam gic to thnh u tha
mn bi.
- Vi mt gi tr n > 4 bt k, ta s chng minh rng :

1
( ) 1
2
n
S n S
| + | (
+
|
(
\
.
Gi a l s nh nht c gn cho cc on thng trong trng hp c n im. Trong
trng hp ti thiu, khng mt tnh tng qut, ta gi s rng a = 1, ta gi hai u mt ca on
thng no c gn s 1 l X v Y.
Trong n 2 im cn li, nu c mt im c ni vi X v Y bi mt on thng gn
bi s 1 th im cng vi X v Y s to thnh mt tam gic u khng tha mn bi.
Do , nu gi A l tp hp tt c cc im ni vi X bi mt on thng gn s 1 (c tnh lun
im Y) v B l tp hp tt c cc im ni vi Y bi mt on thng gn s 1 (c tnh lun
im X) th gia A v B khng c phn t no chung hay A B n + = .
*Ta c cc nhn xt sau :
- Nu ly mt im bt k trong tp A v mt im bt k trong B th hai im cng phi
c ni bi on thng gn s 1 v nu khng th hai im s cng vi X s to thnh mt
tam gic khng tha mn bi (tam gic hoc khng c hai s c gn trn hai cnh bng
nhau hoc c hai cnh bng nhau nhng cnh cn li gn s 1 nh hn).
- Hai im bt k trong A c ni vi nhau bi mt on thng gn s ln hn 1 bi nu
khng th khi chn thm mt im trong B, ta s c mt tam gic khng tha mn bi (tam
gic u). Tng t vi tp hp B. Tc l trong cc tp A v B u c cha cc s ln hn 1.
Tip theo, ta li thy trong mi tp A, B nh vy u cn thm ( ), ( ) S A S B s na
gn cho cc on thng. Gi s A B th
1 1
1
2 2
n n
A
+ ( (
+ =
( (

.
2
2
1
1
1
1
www.VNMATH.com
25

Ta hon ton c th gn cc s tp A trng vi cc s tp B nn cc s cn c thm
na l
1
2
n
S
| + | (
|
(
\
, tnh thm s 1 nh nht c gn cho on XY ban u, ta c:
1
( ) 1
2
n
S n S
| + | (
+
|
(
\
.
T , p dng lin tip kt qu ny, ta c: (ch rng S(4) = 2).
(2006) 1 (1003) 2 (502) ... 9 (4) 11 S S S A + + + = .
Tip theo, ta s chng minh rng gi tr 11 ny tha mn bi.
* Tht vy :
Ta xy dng cch gn cc im t thp n cao bng cch ghp cc b im t hn li. C th
nh sau :
- u tin ta xy dng cho b 4 im. Cch gn tng t nh trn, nhng trong trng hp
ny gn 4 on bi s 11 v 2 on bi s 10.
- Ghp 2 b ny li v tch ra t mt trong hai b ra 2 im, gn cho on thng ni 2 im
bi s 10, ta c tt c 8 im.
- Tip tc ghp tng t nh vy theo th t nh sau :
4 8 16 32 63 126 251 502 1003 2006
(Cc trng hp t 32 n 63 hoc tng t ta phi b i 1 im no mt trong hai b
ra ngoi). Mi ln ghp hai b im li th s gn trn on c tch ra li gim i 1 n v,
n khi ghp c 2006 im th s chnh l 1.
D thy cch gn s cho cc on thng ny tha mn bi.
Vy gi tr nh nht ca s tt cn tm l 11.
www.VNMATH.com
26

Bi 4. Chng minh rng vi mi s thc | | , , 1; 2 x y z , ta lun c bt ng thc sau :
1 1 1
( )( ) 6( )
x y z
x y z
x y z y z z x x y
+ + + + + +
+ + +
.
Hi ng thc xy ra khi v ch khi no ?

Trc ht ta thy rng :
2
1 1 1 ( )
( )( ) 9
x y
x y z
x y z xy

+ + + + =

,
2
( )
6( ) 9
( )( )
x y z x y
y z z x x y y z z x

+ + =
+ + + + +

.
Ta cn chng minh :
2 2
1 1 1 ( ) 3( )
( )( ) 6( )
( )( )
x y z x y x y
x y z
x y z y z z x x y xy y z z x

+ + + + + +
+ + + + +


vi mi s thc x, y, z thuc on [1 ; 2].
t
1 3
( )( )
x
S
yz x y x z
=
+ +
,
1 3
( )( )
y
S
zx y x y z
=
+ +
,
1 3
( )( )
z
S
xy z x z y
=
+ +
.
Bt ng thc cho vit di dng tng ng l:
2 2 2
( ) ( ) ( ) 0
x y z
S y z S z x S x y + + .
Khng mt tnh tng qut, ta gi s 2 1 x y z .
Ta s chng minh rng , 0
x y
S S . Tht vy:
2
0 2 0
x
S x xy xz yz + + , ng.
2
0 2 ( ) ( ) 0
y
S y yx yz zx x y z z z y x + + + + (do | | , , 1; 2 x y z nn 0 y z x + ).
- Nu 0
z
S , ta c pcm.
- Nu 0
z
S < , ta chng minh c rng 2 0, 2 0
x z y z
S S S S + + .
Khi d dng thy rng v:
2 2 2
( ) 2 ( ) ( ) x y y z z x ( +

v 0
z
S < nn
2 2 2 2 2
( ) ( ) ( ) ( 2 )( ) ( 2 )( ) 0
x y z x z y z
S y z S z x S x y S S y z S S z x + + + + +
Vy trong mi trng hp, ta lun c pcm.
ng thc xy ra khi x = y = z hoc y = z = 1, x = 2 v cc hon v ca chng.
www.VNMATH.com
27

Bi 5. Cho tam gic ABC l tam gic nhn, khng cn, ni tip trong ng trn tm O bn
knh R. Mt ng thng d thay i sao cho d lun vung gc vi OA v lun ct cc tia AB,
AC. Gi M, N ln lt l giao im ca ng thng d v cc tia AB, AC. Gi s cc ng
thng BN v CN ct nhau ti K; gi s ng thng AK ct ng thng BC.
1. Gi P l giao ca ng thng AK v ng thng BC. Chng minh rng ng trn
ngoi tip ca tam gic MNP lun i qua mt im c nh khi d thay i.
2. Gi H l trc tm ca tam gic AMN. t BC = a v l l khong cch t im A n ng
thng HK. Chng minh rng ng thng HK lun i qua trc tm ca tam gic ABC.
T suy ra:
2 2
4 l R a . ng thc xy ra khi v ch khi no?












1. Khng mt tnh tng qut, gi s AB < AC (trng hp cn li hon ton tng t).
Do tam gic ABC khng cn nn AO khng vung gc vi BC v MN khng song song vi BC,
do MN phi ct ng thng BC ti mt im, gi s l Q; gi I l trung im BC.
Theo nh l Menelaus cho ba im Q, M, N thng hng: . . 1
NA MB QB
NC MA QC
= .
Mt khc, theo nh l Cva cho cc on AP, BN, CM ng quy, ta c: . . 1
NA MB PB
NC MA PC
= .
T , suy ra:
PB QB
PC QC
= hay Q, B, P, C l mt hng im iu ha, suy ra:
2 2
. IP IQ IB IC = =
Do I l trung im BC nn
2 2 2 2
OI BC QI BI OQ OB = , do :
2 2 2 2 2
. . . QI QP QI QI PI QI IB OQ OB QBQC = = = =
(do theo tnh cht phng tch ca Q i vi (O) th
2 2 2 2
. OQ OB OQ R QBQC = = ).
M t gic BMNC cng ni tip v c

NCB xAB AMN = = (vi Ax l tia tip tuyn ca (O)).
Suy ra . . QM QN QB QC = .
Q I P
K
N
M
O
A
B
C
www.VNMATH.com
28

T suy ra . . QM QN QP QI = , suy ra t gic MNIP ni tip hay ng trn ngoi tip tam gic
MNP lun i qua im I c nh. Ta c pcm.
2. Gi BD, CE l hai ng cao ca tam gic ABC, L l trc tm ca tam gic ABC; gi MF,
NG l hai ng cao ca tam gic AMN, H l trc tm ca tam gic AMN. Ta cn chng
minh rng H, K, L thng hng.















Xt ng trn (O
1
) ng knh BN v (O
2
) ng knh CM.
Ta thy: KM.KC = KB. KN nn K c cng phng tch n (O
1
), (O
2
), tc l K thuc trc ng
phng ca hai ng trn ny.
ng thi, d thy rng cc im D, G thuc (O
1
) v M, F thuc (O
2
).
Do H, L l trc tm ca tam gic ABC v AMN nn LB. LD = LC. LE, HN. HG = HE. HM; tc
l H, L cng thuc trc ng phng ca hai ng trn (O
1
), (O
2
).
T suy ra H, K, L cng thuc trc ng phng ca (O
1
), (O
2
) nn chng thng hng.
T suy ra l AL .
Mt khc do tam gic ABC nhn nn
2
2 2 2
2 4
4
BC
AL OI R R a = = = .
Do
2 2
4 AL l R a = . y chnh l pcm.
n y, ta s tm v tr ca d sao cho ng thc xy ra.
Gi s d ct AB, AC ti M v N tha mn .
AN AM
k MN k BC
AB AC
= = = .
F
G
H
L
E
D
Q I P
K
N
M
O
A
B
C
www.VNMATH.com
29

Gi R, S ln lt l trung im ca BN v CM; suy ra R, S cng chnh l tm ca hai
ng trn (O
1
), (O
2
).
Ta thy khi ng thc xy ra th AL vung gc vi trc ng phng ca (O
1
), (O
2
), tc
l AL song song vi ng ni tm RS ca hai ng trn ny, m AL vung gc vi BC nn
RS phi vung gc vi BC.
Ta c: 2RS BC NM = +

, m . 0 ( ). 0 RS BC BC NM BC = + =

. Do gc to bi MN v BC
chnh l

MQB ANM ACB ABC ACB = = nn t ng thc trn suy ra:
2
1
. . .cos( )
cos( )
BC BC MN BC kBC B C k
B C
= = =


.
Vy ng thc xy ra khi v ch khi
1
cos( )
k
B C
=

, tc l ng thng d ct AB ti M,
AC ti N sao cho
1
cos( )
AN AM
AB AC B C
= =

.
www.VNMATH.com
30

Bi 6. Cho dy s thc ( )
n
a c xc nh bi:
0 1
1 1
1, ( )
2 3
n n
n
a a a
a
+
= = + vi mi n = 1, 2, 3,
Chng minh rng vi mi s nguyn n, s
2
3
3 1
n
n
A
a
=

l mt s chnh phng v n c t
nht n c nguyn t phn bit.

Trc ht, ta s chng minh rng
1
3 4 ( 3)
n n n
A A A
+
= + vi mi n nguyn dng. (*)
Tht vy:
Ta c:
2
2 2
9 3
3 3
3 1 3 1
n
n
n n
a
A
a a
+ = + =

nn
( )
2 2
2 2
2
9 108
4 ( 3) 36 .
3 1
3 1
n n
n n n
n
n
a a
A A A
a
a
+ = =


.
Mt khc:

( )
2
1 2 2
2
2
1
2
108 9 9
3
1 1
3 1
3 1
3. ( ) 1
2 3
n
n
n
n
n
n
a
A
a
a
a
a
+
+
= = =


+
.
Do : (*) c chng minh, tc l
1
3 4 ( 3)
n n n
A A A
+
= + vi mi n nguyn dng.
Hn na, ta tnh c: 9
n
A = nn d dng thy rng
n
A chia ht cho 3 vi mi n.
Tip theo, ta s chng minh bng quy np rng: 1
3
n
A
+ l s chnh phng vi mi n (**).
- Vi n =1,
1
1 4
3
A
+ = l mt s chnh phng nn (**) ng.
- Gi s vi n = k, (**) cng ng, tc l tn ti s t nhin p sao cho
2
1
3
k
A
p + = .
Ta c:
2 2 2 2 1
4 ( 3)
1 1 4. .(1 ) 1 4 ( 1) 1 (2 1)
3 9 3 3
k k k k k
A A A A A
p p p
+
+
+ = + = + + = + = cng l
mt s chnh phng. Suy ra (**) cng ng vi 1 n k = + .
Do , (**) c chng minh.
T
1 1
3 4 ( 3) 4 .( 1)
3
n
n n n n n
A
A A A A A
+ +
= + = + v (**), ta cng d dng chng minh c bng
quy np rng
n
A l s chnh phng vi mi n.

Cui cng, ta s chng minh rng
n
A c t nht n c nguyn t i mt khc nhau cng bng
phng php quy np. (***)
- Vi n = 1, r rng (***) ng.
- Gi s (***) ng vi n = k, tc l
k
A c t nht k c nguyn t khc nhau.
Ta xt hai trng hp:
www.VNMATH.com
31

+ Nu k c t nht k + 1 c nguyn t khc nhau th r rng (***) ng.
+ Nu k c ng k c nguyn t i mt khc nhau, gi s l
1 2 3
, , ,..,
k
p p p p .
Khi : ( , ), 1,
k i
A p i k = l 1 hoc 3.
Gi s
1 k
A
+
ch c ng k c nguyn t i mt khc nhau l cc gi tr trn th cn phi c
*
3 3 , , 2
m
k
A m m + = .
Nhng khi th 3(mod 9)
k
A khng phi l s chnh phng, mu thun.
T dn n
1 k
A
+
phi c mt c nguyn t no khc k c c, tc l c t nht k + 1 c
nguyn t i mt khc nhau hay (***) ng vi 1 n k = + .
Do (***) c chng minh.
Vy vi mi n nguyn dng, s
2
3
3 1
n
n
A
a
=

l mt s chnh phng v n c t nht n


c nguyn t phn bit, bi ton c gii quyt hon ton.

www.VNMATH.com
32

LI GII THI CHN I TUYN QUC GIA
D THI IMO 2007

Bi 1.
Cho hai tp hp A,B l tp hp cc s nguyn dng tha mn A B n = = (vi n l s
nguyn dng) v c tng cc phn t bng nhau. Xt bng vung n n .
Chng minh rng ta c th in vo mi vung ca bng mt s nguyn khng m
tha mn ng thi cc iu kin:
i/ Tp hp tng cc s mi hng l tp A.
ii/ Tp hp tng cc s mi ct l tp B.
iii/ C t nht
2
( 1) n k + s 0 trong bng vi k l s cc phn t chung ca A v B.

Trc ht, ta thy rng nu mt gi tr k sao
cho tn ti 2 phn t bng nhau mi tp l
k k
a b t = = th ta in s t vo vung nm hng th
k v ct th k, cc cn li ca hng th k v ct th k
u in vo s 0; nh th th tng cc s hng v ct
ny tha mn bi v khng nh hng n cc hng
v ct khc. Do , khng mt tnh tng qut, ta xt
trng hp A B = (trng hp c cc phn t
chung th in thm vo cc hng v ct theo cch
tng t nh trn), tc l s phn t chung ca hai tp
l 0 k = .
Ta s chng minh bi ton ny bng quy np. Gi l tp hp cc iu kin i/, ii/, iii/
nh trn (iu kin iii/ tng ng vi trng hp xt s nguyn dng n).
Vi n = 1, bi ton hin nhin ng.
Gi s bi ton ng vi mi s tp hp c n 1 phn t. Ta s chng minh rng vi hai
tp A, B c n phn t, ta cng c th xy dng mt bng n n tha mn iu kin .
Tht vy, xt hai tp hp
1 2 3 1 2 3
{ , , ,..., }, { , , ,..., }
n n
A a a a a B b b b b = = trong :
1 2 3
...
n
a a a a < < < < ,
1 2 3
...
n
b b b b < < < < (hai tp ny khng c phn t no chung).
Gi s
1 1
a b < . Do tng cc phn t hai tp bng nhau nn tn ti mt ch s i tha mn
1 1 1 1 1
( ) 0
i i
a b b a a b a > > > . Ta xt hai tp hp A*, B* nh sau:
2 3 1 1 1
* { , ,..., , ,..., }
i i n
A a a a a b a a

= + ,
2 3
* { , ,..., }
n
B b b b = .
0
0 0
0
0
0 0
0
0
0
0
0
n
i
i
2
1
n
2
1
www.VNMATH.com
33

Hai tp hp ny c cng s phn t l n 1 nn
theo gi thit quy np, tn ti mt bng c kch thc
( 1) ( 1) n n tha mn iu kin (trong bng ny c
t nht
2
( 2) n s 0).
Ta thm vo bn tri bng mt ct v bn trn bng mt
hng na nh hnh v. gc bn tri v pha trn, ta
in s
1
a , hng th i ca bng ban u (hng c tng
cc phn t bng
1 1 i
a b a + ), ta in s
1 1
b a ; cn tt
c cc cn li ca hng v ct va thm vo, ta in
vo cc s 0. Khi , bng ny c tng cc phn t mi
hng l tp A v tng cc phn t mi ct l tp B, s cc s 0 bng va lp c khng nh
hn
2 2
( 2) 2( 1) 1 ( 1) n n n + = v do n tha mn iu kin .
Do , bi ton cng ng vi mi tp hp c n phn t.
Theo nguyn l quy tp, bi ton ny ng vi mi s nguyn dng n.
Vy ta c pcm.
n - 1
i
2
1
n - 1 3 2 1
www.VNMATH.com
34

Bi 2.
Cho tam gic nhn ABC vi ng trn ni tip I. Gi ( )
a
k l ng trn c tm nm
trn ng cao ca gc A v tip xc trong vi ng trn (I) ti
1
A . Cc im
1 1
, B C xc
nh tng t .
1/ Chng minh
1 1 1
, , AA BB CC ng qui ti P.
2/ Gi ( ), ( ), ( )
a b c
J J J ln lt l cc ng trn i xng vi ng trn bng tip cc
gc A, B, C ca tam gic ABC qua trung im BC, CA, AB.
Chng minh P l tm ng phng ca 3 ng trn ni trn.

1/ Trc ht, ta s chng minh b sau:
Cho tam gic ABC ngoi tip ng trn (I) c D l tip im ca ng trn bng tip
gc A ln BC. Gi M, N l giao im ca AD vi (I) (N nm gia A v M). Gi s IM ct ng
cao AH ti K. Chng minh rng: KA = KM.
* Tht vy:
Gi E l tip im ca (I) ln
BC. Gi s IE ct (I) ti im th hai l
N khc E. Qua N v ng thng
song song vi BC ct AB v AC ln
lt ti B v C. D thy tn ti mt
php v t bin tam gic ABC thnh
tam gic ABC. Php v t cng bin
tip im N ca ng trn bng tip
(I) ca ABC ln BC thnh tip
im D ca ng trn bng tip (J)
ca ABC ln BC. Suy ra A, N, D
thng hng hay N trng vi N. Khi ,
tam gic IMN ng dng vi KMA
(do IN // AK), m IMN cn ti I nn
KAM cn ti K hay KA = KM.
Ta c pcm.
T y suy ra: ng trn c tm
thuc ng cao gc A, i qua A v
tip xc vi (I) ti M th M thuc AD.
D thy ng trn l duy nht.
*Tr li bi ton:
Gi D, E, F ln lt l tip im ca ng trn bng tip cc gc A, B, C ca tam gic
ABC ln cc cnh BC, CA, AB. Theo b trn, ta thy:
1 1 1
, , A AD B CF C BE .
Suy ra:
1 1 1
, , AA BB CC ng quy khi v ch khi AD, BE, CF ng quy. (1)
C'
B'
N
K
H
M
E
D
J
I
A
B
C
www.VNMATH.com
35

Mt khc: nu ta t
, , ,
2
BC a CA b AB c
AB BC CA
p
= = =
+ +
=

th c th d dng tnh c:
DB EC p c = =
DC AF p b = =
AE BF p a = = .
Suy ra: . . 1
DB EC FA
DC EA FB
= ,
theo nh l Ceva o, ta c
AD, BE, CF ng quy. (2)
T (1) v (2), ta c
1 1 1
, , AA BB CC ng quy.
Ta c pcm.

2/ Gi A, B ln lt l
trung im ca BC, CA;
A
2
, B
2
, C
2
ln lt l tm
ng trn bng tip cc
gc A, B, C ca ABC .
Gi D, E ln lt l tip
im ca (I) ln BC, CA.
D thy D i xng vi D
qua trung im A ca BC,
A
2
i xng vi J
a
qua A
nn J
a
D // A
2
D, m
2
'
a
A D BC J D BC .
Do : (J
a
) tip xc vi BC
ti D.
Hon ton tng t:
(J
b
) tip xc vi CA ti E.
Ta c:
' ' CD CE = nn phng tch
t C n (J
a
) v (J
b
) bng
nhau, tc l C thuc trc
ng phng ca hai ng trn ny.
C
1
B
1
E
C
2
I
A
1
P
F
D
A
2
A
B
C
B
2
A'
B'
E'
C
2
J
b
B
2
J
a
I
P
F
A
2
D'
A
B
C
www.VNMATH.com
36

Ta s chng minh rng CP, cng chnh l CF, vung gc vi on ni tm J
a
J
b
ca hai
ng trn (J
a
), (J
b
).
Theo cch xc nh cc im J
a
, J
b
, ta thy A l trung im ca A
2
J
a
, B l trung im ca B
2
J
b
.
Do :
2 2
2 ' '
a b
A B A B J J = +

hay
2 2
2 ' '
a b
J J A B A B = =

2 2
BA A B

. Ta cng c:
2 2
CF CC C F = +

.
Ta c:
2 2 2 2 2 2 2 2 2 2 2 2
. ( )( ) . . . .
a b
J J CF BA A B CC C F BACC BAC F A B CC A B C F = + = + =


2 2 2 2
. . BACC A B C F =

(do C
2
F vung gc vi AB, A
2
B
2
vung gc vi CC
2
). (1)
Mt khc, ta thy A, B, C chnh l chn cc ng cao ca tam gic A
2
B
2
C
2
nn r rng:
2 2 2 2
C AB C B A , m C
2
F l ng cao ca
2
C AB , C
2
C l ng cao ca
2 2 2
C B A nn:
2 2
2 2
C F C C
AB A B
=
2 2 2 2
. . C F A B ABCC = . Cng t hai tam gic
2
C AB ,
2 2 2
C B A ng dng; ta c:
2 2 2 2
( , ) ( , ) BA CC A B C F =

. Do :
2 2 2 2
. . BACC A B C F =

. (2)
T (1) v (2), suy ra:
2
. 0
a b
J J C F =

hay
2 a b
C F J J .
Do C
2
F chnh l trc ng phng ca hai ng trn (J
a
), (J
b
), tc l P thuc trc
ng phng ca hai ng trn (J
a
), (J
b
).
Hon ton tng t, ta cng c P thuc trc ng phng ca hai ng trn (J
c
), (J
b
).
T suy ra P chnh l tm ng phng ca (J
a
), (J
b
), (J
c
).
y chnh l pcm.


www.VNMATH.com
37

Bi 3.
Cho tam gic ABC. Tm gi tr nh nht ca biu thc sau:

2 2 2 2 2 2
2 2 2
cos cos cos cos cos cos
2 2 2 2 2 2
cos cos cos
2 2 2
A B B C C A
S
C A B
= + +
* Trc ht, ta s chng minh b :
Vi mi a, b, c khng m v khng ng thi bng 0, ta c:

2 2 2
1 1 1 9
( ) ( ) ( ) 4( ) a b b c c a ab bc ca
+ +
+ + + + +
.
Du ng thc xy ra khi a = b = c hoc a = b, c = 0 v cc hon v.
* Chng minh:
Quy ng v khai trin BT trn, ta cn chng minh rng:
5 4 2 3 3 4 3 2 2 2 2
(4 3 ) ( 2 ) 0
sym sym
a b a b a b a bc a b c a b c + +

(*)
Theo BT Schur cho cc s khng m a, b, c, ta c:
( )( ) 0
sym
a a b a c

, nhn vo hai v cho s abc khng m v khai trin, ta c:


4 3 2 2 2 2
( 2 ) 0
sym
a bc a b c a b c +

. (1)
Hn na, theo BT Cauchy, ta c:
5 4 2 3 3 5 4 2 5 3 3
(4 3 ) ( ) 3( ) 0
sym sym sym
a b a b a b a b a b a b a b = +

. (2)
Cng tng v cc BT (1) v (2) li, ta c BT (*).
ng thc xy ra trong (*) khi v ch khi ng thc xy ra trong (1) v (2), tc l khi
a = b = c hoc a = b, c = 0 v cc hon v. B c chng minh.
* Tr li bi ton:
Ta c:
2
2
1
cos
2
tan 1
2
A
A
=
+
. Tng t vi
2 2
cos , cos
2 2
B C
.
www.VNMATH.com
38

Thay cc bin i ny vo biu thc cho, ta c:
2
2 2
1
( 1)( 1)
x
S
y z
+
=
+ +

, trong tan , tan , tan ; , , 0


2 2 2
A B C
x y z x y z = = = > .
( y k hiu x

l tng i xng ly theo cc bin x, y, z).


Mt khc, trong tam gic ABC, ta lun c:
tan tan tan tan tan tan 1 1
2 2 2 2 2 2
A B B C C A
xy yz zx + + = + + = .
Do :
2
2
2 2 2 2
( )
1 ( )( )
( 1)( 1) ( )( ) ( )( )( )( )
x xy
x x y x z
S
y z y xy z xy y z y x z x z y
+
+ + +
= = = =
+ + + + + + + +



2
1
( ) x y
=
+

.
p dng b trn, ta c:
9 9
4( ) 4
S
xy yz zx
=
+ +
.
Vy gi tr nh nht ca S l
9
4
t c khi v ch khi x y z = = hay ABC l tam gic u.
www.VNMATH.com
39

Bi 4. Tm tt c cc hm s lin tc : f tha mn:

2
1
( ) ( )
3 9
x
f x f x = + + vi mi x.
Ta c:
2 2 2
1 1 1 1 1
( ) ( ) (( ) ),
3 9 6 12 6 12
x
x x f x f x x + + = + + = + + .
t
1 1
6 6
y x x y = + = . Thay vo gi thit cho, ta c:
2
1 1
( ) ( )
6 12
f y f y = + . (*)
Xt hm s: ( ) : g x tha mn:
1 1
( ) ( ) ( ) ( ),
6 6
g x f x f x g x x + = = . (1)
T (*), ta c:
2
1
( ) ( ),
4
g x g x x = + , r rng ( ) g x cng lin tc.
Ta s xc nh hm s ( ) g x tha mn iu kin trn.
Ta thy:
2 2
1 1
, ( ) ( ) (( ) ) ( )
4 4
x g x g x g x g x = + = + = nn ( ) g x l hm s chn.
Ta ch cn xt 0 x . Ta c hai trng hp:
- Vi
0
1
2
x : Xt dy s:
2
1 0 1
1
, , 1
4
n n
u x u u n
+
= = + . Khi :
2
1
1
( ) 0
2
n n n
u u u
+
= nn
dy ( )
n
u tng. Mt khc, bng quy np, ta chng minh c
1
,
2
n
u n , tc l dy ny b chn
trn. T suy ra n c gii hn. Gi t l gii hn th
2
1 1
4 2
t t t = + = .
Do :
0
1 1
( ) ( ), 0;
2 2
g x g x
(
=
(

.
- Vi
0
1
2
x > : Tng t nh trn, xt dy s
1 0 1
1
, 0, 1
4
n n
v x v v n
+
= = .
Khi :
2
1
1
( )
1
2
0
4 1
4
n
n n n n
n n
v
v v v v
v v
+

= = <
+
nn dy cho l dy gim. Cng bng quy
np, ta chng minh c
1
2
n
v > , tc l dy cho b chn di, suy ra n c gii hn.
Gi k l gii hn th
1 1
4 2
k k k = = .
Do :
0
1 1
( ) ( ), ( ; )
2 2
g x g x = + . t
1
( )
2
g a = .
T suy ra: ( ) , 0 g x a x = hay ( ) , g x a x = . (2)
T (1) v (2), ta c: ( ) , f x a x = . y l tt c hm s cn tm.
www.VNMATH.com
40

Bi 5.
Cho A l tp con cha 2007 phn t ca tp:{ } 1, 2, 3,..., 4013, 4014 tha mn vi mi
, a b A th a khng chia ht cho b. Gi m
A
l phn t nh nht ca A.
Tm gi tr nh nht ca m
A
vi A tha mn cc iu kin trn.

Chia tp hp {1, 2, 3, ..., 4013, 4014} thnh 2007 phn
1 2 2007
... P P P (mi tp hp
cha t nht mt phn t ca { } 1, 2, 3,..., 4013, 4014 ) tha mn tp hp
a
P cha tt c cc s
nguyn dng c dng 2 (2 1)
n
a , trong n l mt s khng m. Khi , tp hp con A ca
{ } 1, 2, 3,..., 4013, 4014 khng th cha hai phn t cng thuc mt trong 2007 tp hp v nu
khng th r rng c mt s s chia ht cho s cn li, mu thun. Mt khc, A li c ng 2007
phn t nn A cha ng 1 phn t ca mi tp
a
P ni trn.
Gi
i
l mt phn t ca A vi , 1, 2007
i i
P i = . Xt cc phn t
1 2 5 1094
, , ,... ln
lt c cc dng
2 7
2 , 3.2 , 3 .2 ,..., , 3 .2
n n n n
; r rng mi phn t ch c hai c nguyn t l 2 v
3. Ta cng thy rng ly tha ln nht ca 2 trong
1
phi ln hn ly tha ln nht ca 2 trong
2
v nu ngc li th
2 1
, mu thun.
Hon ton tng t vi cc phn t khc trong dy
1 2 5 1094
, , ,... , tc l nu i j < th
ly tha ca 2 trong
i
phi ln hn ly tha ca 2 trong
j
. Do , ly tha ca 2 trong dy
1 2 5 1094
, , ,... l mt dy gim thc s. Do c 8 phn t trong dy trn (tng ng vi ly
tha ca 3 thay i t 0 n 7) nn gi tr ca
1
t nht l
7
2 128 = . Hn na, cc phn t ca
dy c gi tr ca ly tha 3 tng dn (nu ngc li, gi tr ca ly tha 3 gim m ly tha 2
cng gim nn c mt s chia ht cho s khc, mu thun).
Suy ra:
1
chnh l gi tr nh nht trong dy trn.
Hon ton tng t vi cc s nguyn t khc, ta cng xt s c dng 2 .3 ...
i i k
n n n
i k
p = ,
trong 2, 3, , p
k
l cc c nguyn t khng vt qu 4014. T suy ra
1
cng chnh l
phn t nh nht trong cc phn t ca tp A. Do phn t nh nht m
A
ca tp A chnh l
1
,
nh chng minh trn
1
128 hay 128
A
m .
Ta s chng minh rng 128 chnh l gi tr nh nht ca m
A
bng cch ch ra mt tp hp
con A ca { } 1, 2, 3,..., 4013, 4014 tha mn iu kin bi.
Xt tp hp:
{ }
( ) ( )
2 (2 1) | 1338 3 .(2 1) 4014
f i f i
i
A i i = = .
www.VNMATH.com
41

R rng tp hp ny c ng 2007 phn t thuc { } 1, 2, 3,..., 4013, 4014 .
Ta s chng minh rng
x
khng chia ht cho
y
, vi x y > (tc l tp hp ny tha mn bi).
Tht vy, gi s ngc li tn ti x, y tha mn
x y
, khi :
( ) ( )
2 (2 1) 2 (2 1)
f x f y
x y ,
tc l ( ) ( ) f x f y v (2 1) (2 1) x y .
T cch xc nh cc gi tr u, v; ta c: (2 1) (2 1) 2 1 3(2 1) x y x y , ng thi
( ) 1 ( ) ( ) 1
3 (2 1) 4014 3 (2 1) 3 (2 1) ( ) ( )
f y f x f x
y x y f y f x
+ +
> > . Mu thun ny chng t tt
c cc phn t ca A u tha mn vi mi , a b A th a khng chia ht cho b.
Vy 128 chnh l gi tr nh nht ca m
A
cn tm.
www.VNMATH.com
42

Bi 6.
Cho a gic 9 cnh u (H). Xt ba tam gic vi cc nh l cc nh ca a gic (H)
cho sao cho khng c hai tam gic no c chung nh.
Chng minh rng c th chn c t mi tam gic 1 cnh sao cho 3 cnh ny bng nhau.

K hiu hnh (H) cho l a gic
1 2 3 8 9
... A A A A A nh hnh v. Trc ht, ta thy
rng di cc cnh v cc ng cho ca
hnh (H) ch thuc 4 gi tr khc nhau (nu
gi R l bn knh ng trn ngoi tip ca
(H) th ta d dng tnh c cc gi tr l
2 .sin
9
R

,
2
2 sin
9
R

,
3
2 sin
9
R

,
4
2 sin
9
R

)
ta t chng l
1 2 3 4
, , , a a a a theo th t tng
dn ca di. R rng cc tam gic c nh
thuc cc nh ca (H) s c cnh c di
thuc 1 trong 6 dng sau:
1 1 2 2 2 4 1 3 4
3 3 3 2 3 4 4 4 1
( , , ), ( , , ), ( , , ),
( , , ), ( , , ), ( , , )
a a a a a a a a a
a a a a a a a a a
.
Gi s 3 tam gic c ly ra l
1 2 3
, , .
Xt cc trng hp sau:
- Nu trong cc tam gic c mt tam gic u, r rng, tam gic ny phi c di cc
cnh l
3
2 .sin
9
R

; khng mt tnh tng qut, gi s l tam gic
1 4 7
A A A . Do cc tam gic
1 2 3
, , khng c hai nh no trng nhau nn ta s lp mt tam gic c cc nh l mt trong
hai nh ca cc tp hp
2 3 4 5 7 8
{ , },{ , },{ , } A A A A A A . Ta s chng minh rng tam gic phi c t
nht 1 cnh c di l
3
2 .sin
9
R

, tc l hai nh c ch s c cng s d khi chia cho 3. Gi s
ngc li, trong hai tam gic cn lp, khng c tam gic no c cnh l
3
2 .sin
9
R

, khi nh
A
2
phi ni vi A
4
v A
4
phi ni vi A
8
, nhng khi A
8
c ni vi A
2
l hai nh c ch s
chia cho 3 cng d l 2, mu thun. Do , trong hai tam gic lp c, lun c mt cnh c
di l
3
2 .sin
9
R

. Suy ra trng hp ny lun c tam gic tha mn bi.
a
1
a
2
a
3
a
4
9
8
7
6
5
4
3
2
1
A
A
A
A
A
A
A
A
A
www.VNMATH.com
43

- Nu trong cc tam gic , khng c tam gic no u . Khi cc tam gic c xt
khng c ba nh cng thuc mt trong ba tp hp sau:
1 1 4 7
{ , , } A A A = ,
2 2 5 8
{ , , } A A A = ,
3 3 6 9
{ , , } A A A = . Ta thy mt on thng ni hai im bt k thuc hai tp khc nhau s nhn 1
trong 3 gi tr l
1 2 4
, , a a a . Hn na, khng c tam gic no c di 3 cnh l
1 2 4
( , , ) a a a nn ta
c hai nhn xt:
(1) Mt tam gic c cc nh thuc c ba tp
1 2 3
, , ni trn th s c hai cnh no c
di bng nhau (cc cnh ca n c th l
1 1 2
( , , ) a a a ,
2 2 4
( , , ) a a a ,
4 4 1
( , , ) a a a ) tc l n phi cn.
(2) Mt tam gic c hai trong ba nh thuc cng mt tp th tam gic cc cnh c di
l
2 3 4
( , , ) a a a hoc l
1 3 4
( , , ) a a a , tc l tam gic khng cn.
* Ta xt tip cc trng hp (cc tam gic xt di y l cn nhng khng u):
+ C hai tam gic cn v mt tam gic khng cn: khi theo nhn xt (1), hai tam gic cn
phi c nh thuc cc tp hp khc nhau trong ba tp
1 2 3
, , ; khi , r rng tam gic cn li
cng phi c nh thuc cc tp hp khc nhau, tc l n phi cn, mu thun.
Vy trng hp ny khng tn ti.
+ C mt tam gic cn v hai tam gic khng cn: khi theo nhn xt (2), hai tam gic khng
cn phi c hai nh thuc cng mt tp hp v nh cn li thuc tp hp khc, gi s mt
tam gic c hai nh thuc
1
v mt nh thuc
2
; r rng tam gic khng cn cn li phi c
hai nh thuc
2
, mt nh thuc
3
, suy ra tam gic cn li c hai nh thuc
3
, mt nh
thuc
1
nn n l tam gic cn, mu thun.
Vy tng t nh trn, trng hp ny khng tn ti.
+ C ba tam gic u khng cn: khi theo nhn xt (2), tam gic thuc mt trong hai dng
2 3 4
( , , ) a a a hoc l
1 3 4
( , , ) a a a , tc l cc tam gic ny lun cha 1 cnh c di l a
3
.
Trong trng hp ny, bi ton c gii quyt.
+ C ba tam gic u cn: khi , cc tam gic c di l
1 1 2
( , , ) a a a ,
2 2 4
( , , ) a a a ,
4 4 1
( , , ) a a a .
R rng khng tn ti trng hp c di cc cnh ln lt nhn c ba gi tr nh ba b trn
nn phi c hai b trng nhau, tc l c t nht hai tam gic cn bng nhau v mt tam gic cn
nhn mt trong ba gi tr thuc mt trong cc b trn lm cnh, khi lun c th chn t tam
gic ny mt cnh bng vi cnh y hoc cnh bn ca hai tam gic cn bng nhau kia.
Trong trng hp ny, bi ton cng c gii quyt.
Vy trong mi trng hp, ta lun c pcm.
www.VNMATH.com
44

LI GII THI CHN I TUYN QUC GIA
D THI IMO 2008

Bi 1. Trong mt phng cho gc xOy. Gi M, N ln lt l hai im ln lt nm trn
cc tia Ox, Oy. Gi d l ng phn gic gc ngoi ca gc xOy v I l giao im ca trung
trc MN vi ng thng d. Gi P, Q l hai im phn bit nm trn ng thng d sao
cho IM IN IP IQ = = = , gi s K l giao im ca MQ v NP.
1. Chng minh rng K nm trn mt ng thng c nh.
2. Gi d
1
l ng thng vung gc vi IM ti M v d
2
l ng thng vung gc vi IN ti
N. Gi s cc ng thng d
1
, d
2
ct ng thng d ti E, F. Chng minh rng cc ng
thng EN, FM v OK ng quy.

1. Xt trng hp cc im M, Q v N, P nm cng pha vi nhau so vi trung trc ca MN.
Khi giao im K ca MP v NQ thuc cc on ny.:
Gi I l giao im ca d vi ng trn ngoi tip MON . Do d l phn gic ngoi ca

MON nn I chnh l trung im ca cung

MON , do : IM = IN hay I chnh l giao im


ca trung trc MN vi d. T , suy ra: ' I I hay t gic MION ni tip.
Ta c:

NIO NMO = .
Mt khc: do IM = IN = IP = IQ nn t gic MNPQ ni tip trong ng trn tm I, ng knh
PQ

2 PIN PMN = (gc ni tip v gc tm cng chn cung

PN ).
T cc iu trn, ta c:

2 NMO PMN = MP l phn gic trong ca

OMN .
x
y
d
J
F
E
K
P
Q
I
O
M
N
www.VNMATH.com
45

Tng t, ta cng c: NQ l phn gic trong ca

ONM .
Do K l giao im ca MP v NQ nn K chnh l tm ng trn ni tip ca MON , suy ra K
thuc phn gic trong ca

xOy , tc l K thuc mt ng thng c nh (pcm).


- Nu giao im K nm ngoi cc on MP v NQ: ta cng c lp lun tng t v c
c K l tm ng trn bng tip

MON ca tam gic MON , tc l K cng thuc phn gic


trong ca

xOy , l mt ng thng c nh.


Trong mi trng hp, ta lun c pcm.

2. Gi J l giao im ca d
1
v d
2
. Ta thy t gic MINJ ni tip trong ng trn ng
knh IJ. Hn na: MION cng l t gic ni tip nn 5 im M, N, I, J, O cng thuc mt ng
trn. Do : phn gic trong gc

MON i qua trung im ca cung

MJN .
R rng M, N i xng nhau qua trung trc ca MN nn JM = JN, tc l J cng l trung
im ca cung

MON .
T suy ra: J thuc phn gic trong ca

MON hay O, K, J thng hng.


Ta cn chng minh cc on JO, EN v MF trong
JEF
ng quy.
Tht vy:
. .sin sin
.
. .sin sin
OEJ
OFJ
S OE JO JE OJE JE OJE
OF S JO JF OJF JF OJF
= = = .
Trong JEF v MON , ta c :
sin sin
,
sin sin
JE JFE OM ONM
JF JEF ON OMN
= = .
Mt khc :

sin sin
,
sin sin
OJE ONM
OJE OJN ONM OJF OJM OMN
OJF OMN
= = = = = .
Kt hp li, ta c :
sin sin sin
. . .
sin sin sin
OE JFE OM OFN OM OFN OM
OF JEF ON OEM ON OEM ON
= = =

sin .sin
.
.sin sin
OFN OM MOE ME
ON NOF OEM NF
= = .
Do : . 1 . . 1
OE FN OE NF MJ
OF EM OF NJ ME
= = .
Theo nh l Ceva o, ta c OI, EN v MF ng quy. y chnh l pcm.

www.VNMATH.com
46

Bi 2. Hy xc nh tt c cc s nguyn dng m sao cho tn ti cc a thc vi h s
thc ( ), ( ), ( , ) P x Q x R x y tha mn iu kin:
Vi mi s thc a, b m
2
0
m
a b = , ta lun c ( ( , )) P R a b a = v ( ( , )) Q R a b b = .

Vi m l mt s nguyn dng, ta xt cc trng hp :
- Nu m l s chn, t 2 , * m k k = . Suy ra :
2
0
m k
a b b a = = . Khi cn tm k
sao cho cc a thc ( ) ( ) ( ) , , , P x Q x R x y tha mn c hai iu kin :
(1) ( ( , )) , ( ( , )) ,
k k k
P R x x x Q R x x x x = = .
(2) ( ( , )) , ( ( , )) ,
k k k
P R x x x Q R x x x x = = .
Xt a thc mt bin T(x) tha mn : ( , ) ( ),
k
R x x T x x = . Theo gi thit th
( ( )) ( ( , )) ,
k
P T x P R x x x x = = . T suy ra : deg ( ).deg ( ) 1 P x T x = hay deg ( ) deg ( ) 1 P x T x = = .
Gi s ( ) , , , 0 T x ux v u v u = + , ( ) '. ', ', ' , ' 0 P x u x v u v u = + th
'( ) ' , . ' 1, ' ' 0 u ux v v x x u u u v v + + = = + = hay
1
' , ' ( )
v x v
u v P x
u u u

= = = .
Mt khc : ( ( , )) ( ( )) ( ) ( ) ( ),
k
k k k
x u
Q R x x Q T x Q ux v x Q x P x x
u
| |
= = + = = =
|
\
.
Suy ra : ( ( , )) ( ( , )) , ,
k k
Q R a b P R a b b a a b = = tha
2
0
m
a b = . Nhng theo iu kin ban
u th b cng c th l
k
a . Mu thun ny cho thy cc gi tr m trong trng hp ny khng
tha mn bi.
- Nu m l s l. t
2
( , ) ( )
m
P x x S x = . Suy ra :
2
( ( )) , deg ( ).deg ( ) 2 P S x x x P x S x = = .
Nu nh degS(x) = 2 th deg ( ( )) Q S x l s chn, trong khi : deg ( ( )) deg
m
Q S x x m = =
vi m l s l, mu thun. Suy ra : degS(x) = 1, degP(x) = 2.
Mt khc, trong a thc
2
( , )
m
R x x , bc ca n c th t gi tr nh nht l min(2, ) m m
2
deg ( , ) deg ( ) 1
m
R x x S x = = nn m = 1.
Ta s chng minh rng gi tr m = 1 ny tha mn bi bng cch ch ra cc a thc
P(x), Q(x), R(x, y) tha mn bi.
Tht vy :
Xt cc a thc
2
( ) , ( ) , ( , ) P x x Q x x R x y y = = = .
Khi vi m = 1 th ta c quan h ca a vi b chnh l :
2
a b = . Suy ra :
( , ) R a b b = ,
2
( ( , )) ( ) P R a b P b b a = = = , ( ( , )) ( ) Q R a b P b b = = , tha mn bi.
Vy tt c cc gi tr m tha mn bi l m = 1.
www.VNMATH.com
47

Bi 3. Cho s nguyn 3 n > . K hiu T l tp hp gm n s nguyn dng u tin.
Mt tp con S ca T c gi l tp khuyt trong T nu S c tnh cht: Tn ti s nguyn
dng c khng vt qu
2
n
sao cho vi
1 2
, s s l hai s bt k thuc S ta lun c
1 2
s s c .
Hi tp khuyt trong T c th c ti a bao nhiu phn t ?

Trc ht ta thy rng: Nu S l tp khuyt trong T th tp ' { | } S n x x S = cng l mt
tp khuyt trong T.
Tht vy: Gi s ngc li S khng phi l tp khuyt, khi tn ti hai s nguyn
dng
1 2
' , ' ' s s S sao cho
1 2
| ' ' | s s c = vi c l mt s nguyn dng no khng vt qu
2
n
,
Khi xt tng ng hai phn t
1 1 2 2
' , ' s n s s n s = = th r rng
1 2
, s s S v
1 2 1 2 1 2
| | | ( ' ) ( ' ) | | ' ' | s s n s n s s s c = = = , tc l tn ti hai phn t
1 2
, s s S v
1 2
| |
2
n
s s c = trong khi S l tp khuyt. Mu thun ny suy ra nhn xt trn c chng minh.
Hn na, do | | | ' | S S = nn khi S c s cc phn t l ln nht th tng ng cng c tp
S c s phn t ln nht bng vi S.
T , ta thy c th xt cc tp khuyt S c s cc s phn t khng vt qu
2
n
khng
t hn s cc s phn t ln hn
2
n
. Xt hai tp hp sau:
{ | , }
2
n
A x x S x = , { | , }
2
n
B x x S x = > th , A B A B S = = v theo cch xc nh S
nh trn th A B .
Khi vi c l mt s nguyn dng no khng vt qu
2
n
, ta xt tp hp:
{ | } C x c x A = + . Ta c: | | | | A C = . Do A S nn A cng l mt tp khuyt v khi r rng
, A C B C = = (v nu ngc li th tn ti hai phn t thuc S m hiu ca chng l c,
mu thun).
Suy ra tt c cc phn t thuc tp A hoc B hoc C u l mt s nguyn dng khng
vt qu n, tc l ( ) | | | | | | | | A B C T A B C T n + + = .
Kt hp cc iu ny li, ta c: 2 | | | | A B n + . Do :
4 | | 2 | | 2
| | | |
3 3
A B n
A B
+
+ .
Hn na: , A B A B S = = v | | S l s nguyn nn
2
| | | | | |
3
n
S A B
(
= +
(

.
Do s phn t ca tp khuyt S trong T lun khng vt qu
2
3
n (
(

.
www.VNMATH.com
48

*Ta s ch ra mt tp khuyt tha mn bi c ng
2
3
n (
(

phn t.
Tht vy, xt hai tp hp A, B nh sau:
1
1, 2, 3,...,
3
n
A
+ (
=
`
(
)
, 1, 2,...,
3 3
n n
B n n n
( (
= + +
`
( (
)
v S A B = .
Chn
1
3 2
n n
c
+ (
=
(

. Ta thy:
- Hiu hai phn t bt k trong A khng vt qu
1 1
1
3 3
n n + + ( (
<
( (

.
- Hiu hai phn t bt k trong B khng vt qu
1
( 1) 1
3 3 3
n n n
n n
+ ( ( (
+ = <
( ( (

.
- Hiu mt phn t bt k thuc B vi mt phn t bt k thuc A khng nh hn:
1 1 2 1
( 1) 1
3 3 3 3 3 3
n n n n n n
n n
+ + + + ( ( (
+ + = >
( ( (

.
Khi , r rng S A B = l mt tp khuyt trong T ng vi gi tr
1
3 2
n n
c
+ (
=
(

.
Ta s chng minh rng
2
| |
3
n
S
(
=
(

. T cch xc nh A, B, ta c:
1
| | ,
3 3
n n
A B
+ ( (
= =
( (

.
Ta cn c:
1 2
,
3 3 3
n n n
n
+ ( ( (
+ =
( ( (

(*).
*Xt cc trng hp:
- Nu n chia ht cho 3, tc l n c dng 3 , m m . Suy ra:
1 3 1 3 6 2
2
3 3 3 3 3 3
n n m m m n
m m m
+ + ( ( ( ( (
+ = + = + = = =
( ( ( ( (

.
- Nu n chia 3 d 1, tc l n c dng 3 1, m m + . Suy ra:
1 3 2 3 1 6 6 2 2
2
3 3 3 3 3 3 3
n n m m m m n
m m m
+ + + + ( ( ( ( ( (
+ = + = + = = = =
( ( ( ( ( (

.
- Nu n chia 3 d 2, tc l n c dng 3 2, m m + . Suy ra:
1 3 3 3 2 6 3 6 4 2
1
3 3 3 3 3 3 3
n n m m m m n
m m
+ + + + + ( ( ( ( ( (
+ = + = + + = = =
( ( ( ( ( (

.
T suy ra (*) c chng minh hay tp hp S cho l tp khuyt c
2
3
n (
(

.
Vy gi tr ln nht ca s phn t ca tp khuyt S trong T l
2
3
n (
(

.
www.VNMATH.com
49

Bi 4. Cho m, n l cc s nguyn dng.
Chng minh rng (2 3) 1
n
m+ + chia ht cho 6m khi v ch khi 3 1
n
+ chia ht cho 4m.

Theo khai trin nh thc Newton th:
1
1
(2 3) (2 ) 3 .(2 ) .3 (2 ) 3 (mod 6m)
n
n n n k n k k n n
n
k
m m C m m

=
+ = + + +

.
Do , 6 | (2 3) 1
n
m m+ + 6 | (2 ) 3 1
n n
m m + + 2 | (3 1)
n
m + v 3| (2 ) 1
n
m + .
Cn chng minh rng: 2 | (3 1)
n
m + v 3| (2 ) 1
n
m + (1) 4 | 3 1 (2)
n
m + .
Xt cc trng hp:
* Nu m l s chn:
- Xt iu kin (2):
3 1
n
+
khng chia ht cho 4m v 3 1 2(mod8)
n
+ hoc 3 1 4(mod8)
n
+ , trong khi 4 8 m , tc l
khng th c iu kin (2).
- Xt iu kin (1): t m l s chn, suy ra 3 1 4
n
n + l s l. Ta bit rng: s c dng
3n + 1 ch c c nguyn t l ng d vi 1 modun 4. T , suy ra m tha mn: 2 | (3 1)
n
m +
phi c dng 2(3 1), m k k = + . Suy ra: (2 ) 1 2 .2 .(3 1) 1 2(mod3) (2 ) 1
n n n
m k m + = + + +
khng chia ht cho 3, tc l iu kin (1) cng khng tn ti.
* Nu m l s l:
- Xt iu kin (1): t 2 | (3 1)
n
m + suy ra m khng chia ht cho 3, t 3| (2 ) 1
n
m + suy ra n
phi l s l v nu ngc li th (2 ) 1 2(mod3)
n
m + , mu thun. M n l s l th 3 1 4
n
+ ,
kt hp vi ( , 4) 1 m = , ta c 4 | 3 1
n
m + , y chnh l iu kin (2). Do : (1) (2) .
- Xt iu kin (2): t 4 | 3 1 (4 | 3 1) ( | 3 1)
n n n
m m + + + suy ra n l s l v m c dng
3 1, k k + . Suy ra: (2 ) 1 2 . 1 1 1 0(mod3) 3| (2 ) 1
n n n n
m m m + = + + + ; t (2) ta cng trc
tip c 2 | (3 1)
n
m + . Do : (2) (1) .
Kt hp cc iu trn li, ta c: (1) (2) .
Vy (2 3) 1
n
m+ + chia ht cho 6m khi v ch khi 3 1
n
+ chia ht cho 4m.
y chnh l pcm.
www.VNMATH.com
50

Bi 5. Cho tam gic ABC nhn, khng cn c O l tm ng trn ngoi tip.
Gi AD, BE, CF ln lt l cc ng phn gic trong ca tam gic. Trn cc ng thng
AD, BE, CF ln lt ly cc im L, M, N sao cho
AL BM CN
k
AD BE CF
= = = (k l hng s dng).
Gi (O
1
), (O
2
), (O
3
) ln lt l cc ng trn i qua L, tip xc vi OA ti A ; i qua M tip
xc vi OB ti B v i qua N tip xc vi OC ti C.
1. Chng minh rng vi
1
2
k = , ba ng trn (O
1
), (O
2
), (O
3
) c ng hai im chung v
ng thng ni hai im i qua trng tm tam gic ABC.
2. Tm tt c cc gi tr k sao cho 3 ng trn (O
1
), (O
2
), (O
3
) c ng hai im chung.


Trc ht, ta nu 4 b sau:
(1) Cho ba ng thng i mt phn bit a, b, c v hai ng thng phn bit d, d. Cc
ng thng d, d theo th t ct a, b, c ti
1 1 1 2 2 2
, , ; , , A B C A B C tha mn iu kin:
1 1 2 2
1 1 2 2
A B A B
k
AC A C
= = . Cc im A
3
, B
3
, C
3
thuc a, b, c sao cho:
1 2 1 2 1 2
1 3 1 3 1 3
A A B B C C
A A B B C C
= = .
Khi , A
3
, B
3
, C
3
thng hng v
3 3
3 3
A B
k
A C
= .

(2) Cho ba ng thng phn bit a, b, c v ba ng thng phn bit khc a, b, c . Cc
ng thng a, b, c theo th t ct a, b, c ti
1 1 1 2 2 2 3 3 3
, , ; , , ; , , A B C A B C A B C (cc im ny i
mt phn bit). Khi nu
3 3 1 1 2 2
1 1 2 2 3 3
A B A B A B
AC A C A C
= = th hoc
1 2 1 2 1 2
1 3 1 3 1 3
A A B B C C
A A B B C C
= = hoc a, b, c i
mt song song.

(3) Cho tam gic ABC v M bt k. Cc tia AM, BM, CM ln lt ct BC, CA, AB A
1
, B
1
,
C
1
. Cc ng thng A
1
B
1
, B
1
C
1
, C
1
A
1
ct cc ng thng AB, BC, CA ln lt A
2
, B
2
, C
2
.
Cc im A
3
, B
3
, C
3
theo th t nm trn cc ng thng BC, CA, AB sao cho
1 3 1 3 1 3
1 2 1 2 1 2
, 0
A A B B CC
k k
A A B B CC
= = = . Khi , A
3
, B
3
, C
3
thng hng khi v ch khi 1 k = hoc
1
2
k = .

(4) Cho tam gic ABC khng cn ngoi tip ng trn (I). ng trn (I) tip xc vi cc
cnh BC, CA, AB ln lt ti D, E, F. ng thng EF ct BC ti M, ng thng AD ct (I) ti
N (khc D). Chng minh rng: MN tip xc vi (I).
Cc b (1), (2) c th chng minh d dng bng cc biu din theo vect.
Di y trnh by cc chng minh cho b (3), (4).
www.VNMATH.com
51

*Chng minh b (3).
+ iu kin :
- Vi k = 1, ta c A
3
, B
3
, C
3
theo th t trng vi
A
2
, B
2
, C
2
. V AA
1
, BB
1
, CC
1
ng quy nn theo nh l
Menelaus th
1 1 1
1 1 1
. . 1
A B BC C A
AC B A C B
= . V A
2
, B
1
, C
1
thng
hng nn
2 1 1
2 1 1
. . 1
A B BC C A
A C B A C B
= . Suy ra:
1 2
1 2
A B A B
AC A C
= .
Tng t:
1 2 1 2
1 2 1 2
,
BC B C C A C A
B A B A C B C B
= = .
Nhn tng v cc ng thc trn,
2 2 2 1 1 1
2 2 2 1 1 1
. . . . 1
A B B C C A A B BC C A
A C B A C B AC B A C B
| | | | | |
= =
| | |
\ \ \
.
Tc l A
2
, B
2
, C
2
thng hng hay A
3
, B
3
, C
3
thng hng.
- Vi
1
2
k = , A
3
, B
3
, C
3
theo th t l trung im ca
A
1
A
2
, B
1
B
2
, C
1
C
2
. Theo chng minh trn, ta c:
1 2
1 2
A B A B
AC A C
= . Theo tnh cht t l thc th:
3 1 2 1 2 1 2 1 2 1
1 2 1 2 1 2 1 2 1 3
2
2
A B A B A B AB A B AB A B AB A A
AC A C AC A C AC A C AC A A A C
+
= = = = =
+
Suy ra:
2
3 3 2 1 1
3 2 1 3 1
2
.
2
A B A B A A AB
A C A A A C AC
| |
= =
|
\
. Tng t:
2 2
3 3 1 1
3 1 3 1
,
B C C A BC C A
B A B A C B C B
| | | |
= =
| |
\ \
.
Nhn tng v cc ng thc trn, ta c:
2 2 2
3 3 3 1 1 1
3 3 3 1 1 1
. . . . 1
A B B C C A AB BC C A
A C B A C B AC B A C B
| | | | | |
= =
| | |
\ \ \
. Do , A
3
, B
3
, C
3
th ng hng.
+ iu kin cn: Khi 1 k , ta k hiu
3( ) 3( ) 3( )
, ,
k k k
A B C thay cho A
3
, B
3
, C
3
. Gi s tn ti s k
ng thi khc 1 v
1
2
m
3( ) 3( ) 3( )
, ,
k k k
A B C thng hng. Khi , cc im:
3( ) 3( ) 3( )
, ,
k k k
A B C v
3(1/ 2) 3(1/ 2) 3(1/ 2)
, , A B C i mt khc nhau. D thy:
2 3(1/ 2) 2 3(1/ 2) 2 3(1/ 2)
2 3( ) 2 3( ) 2 3( )
1/ 2 1
1
k k k
A A B B C C
k A A B B C C

= = =

.
B
3
A
3
C
3
C
2
B
2
A
2
A
1
C
1
B
1
A
B C
M
C
2
B
2
A
2
A
1
C
1
B
1
A
B C
M
www.VNMATH.com
52

Theo chng minh iu kin th hai b im A
2
, B
2
, C
2
v
3(1/ 2) 3(1/ 2) 3(1/ 2)
, , A B C thng
hng, m theo iu gi s trn th
3( ) 3( ) 3( )
, ,
k k k
A B C cng thng hng nn theo b (2), hoc
ng thng A
2
B
2
C
2
v
3(1/ 2) 3(1/ 2) 3(1/ 2)
A B C song song hoc
3(1/ 2) 3(1/ 2)
2 2
2 2 3(1/ 2) 3(1/ 2)
A B
A B
A C A C
= .
+ Nu
3(1/ 2) 3(1/ 2)
2 2
2 2 3(1/ 2) 3(1/ 2)
A B
A B
A C A C
= th ch rng:
1 3(1/ 2) 1 3(1/ 2) 1 3(1/ 2)
1 2 1 2 1 2
A A B B C C
A A B B C C
= = , theo b (1) th
A
1
, B
1
, C
1
thng hng, mu thun.
+ Nu A
2
B
2
C
2
v
3(1/ 2) 3(1/ 2) 3(1/ 2)
A B C song
song vi nhau th ch rng
3(1/ 2) 3(1/ 2) 3(1/ 2)
, , A B C theo th t l trung
im ca
1 2 1 2 1 2
, , A A B B C C . Ta c:
3(1/ 2) 3(1/ 2) 1 1 2 2
1
( )
2
A B A B A B = +

,
3(1/ 2) 3(1/ 2) 1 1 2 2
1
( )
2
A C AC A C = +

. Suy ra:
A
1
B
1
song song vi A
2
B
2
v
3(1/ 2) 3(1/ 2)
A B ,
A
1
C
1
song song vi A
2
C
2
v
3(1/ 2) 3(1/ 2)
A C .
T suy ra, A
1
, B
1
, C
1
cng thng
hng, mu thun.
Do ch c 1 k = v
1
2
k = tha mn.
Vy b (3) c chng minh.
*Chng minh b (4).
Gi H l giao im ca
EF v AI. Ta thy: IA EF .
Tam gic AIF vung ti F c
ng cao FH nn :
2 2
. . IF IH IA ID IH IA = = .
Suy ra: ( . . ) IDH IAD c g c .
Do :

IHD IDA = .
Mt khc: tam gic IDN cn ti
I nn

IND IDN IDA = = . T
, ta c:

IND IHD = .
T gic IDNH ni tip. Hn na, t gic IDMH cng ni tip v c

0
90 IDM IHM = = .
Do : 5 im, I, D, M, N, H cng thuc mt ng trn. Suy ra: IMNH ni tip hay

0
90 INM IHM MN IN = = . Vy MN l tip tuyn ca (I). B (4) c chng minh.
A
3(1/2)
A
3(k)
C
2
B
2
A
2
A
1
C
1
B
1
A
B
C
M
C
3(1/2)
B
3(1/2)
B
3(k)
C
3(k)
I
H
M
N
E
F
D
A
B C
www.VNMATH.com
53

*Tr li bi ton.
1. Khi
1
2
k = th L, M, N ln lt l trung im ca cc on AD, BE, CF.
Gi H l trc tm ca ABC v l phng tch ca H i vi ng trn Euler i qua chn 3
ng cao ca ABC . Gi K l giao im ca ng thng AO
1
vi ng thng BC.
Ta s chng minh rng K nm trn (O
1
).
Tht vy:
Do ABC l tam gic nhn nn O nm trong tam gic. Ta c:

0
2 90 AOB ACB OAB ACB = = .
Khng mt tnh tng qut, gi s tia AD nm gia hai tia AO v AB. Khi :




0 0
90 90
2 2
BAC BAC
OAD OAB DAB ACB KAD OAD ACB = = = = +
.
Mt khc:

1
2
ADB DAC DCA BAC ACB = + = + nn

KAD KDA = .
Ta cng c
1 1 1
O A O L AO L = cn ti O
1
nn

1 1
O AL O LA = .
T suy ra:

1
O LA KDA = hay O
1
L // KD, m L l trung im ca AD nn O
1
l trung im ca
AK hay K thuc ng trn (O
1
). Do (O
1
) ct BC ti chn ng cao ca ABC . T suy
ra phng tch ca H i vi ng trn (O
1
) chnh l .
Hon ton tng t vi cc ng trn (O
2
), (O
3
).
1
H
K
O
O
N
M
L
D
E
F
A
B
C
www.VNMATH.com
54

Do H c cng phng tch n cc ng trn (O
1
), (O
2
), (O
3
) nn H chnh l tm ng
phng ca 3 ng trn (O
1
), (O
2
), (O
3
) .
Hn na: do OA l tip tuyn ca (O
1
) ti A nn phng tch ca O i vi (O
1
) chnh l
2
OA . Tng t nh vy, phng tch ca O i vi ng trn (O
2
) v (O
3
) ln lt l
2
OB ,
2
OC , m O l tm ng trn ngoi tip ca ABC nn OA = OB = OC hay O c cng
phng tch n cc ng trn (O
1
), (O
2
), (O
3
), suy ra: O cng l tm ng phng ca 3 ng
trn (O
1
), (O
2
), (O
3
).
Gi s ca 3 ng trn (O
1
), (O
2
), (O
3
) c 3 trc ng phng khc nhau th chng phi
ng quy ti tm ng phng, m O v H cng l tm ng phng ca chng nn O phi trng
vi H hay ABC u, mu thun vi gi thit ABC khng cn.
Do , iu gi s trn l sai v 3 ng trn cho phi c 1 trc ng phng chung,
trc ng phng chnh l ng thng i qua O v H. Ta cng thy rng O nm ngoi c 3
ng trn, H th nm gia cc ng cao ca ABC nn n nm trong c 3 ng trn.
Suy ra ng thng OH ct c 3 ng trn ti 2 im no .
Vy 3 ng trn (O
1
), (O
2
), (O
3
) c ng 2 im chung, hn na, ng thng i qua
hai im chung chnh l ng thng OH v do , n cng s i qua trng tm ca tam gic
(ng thng Euler). Ta c pcm.

2. Ta s chng minh rng ba ng trn (O
1
), (O
2
), (O
3
) c ng hai im chung khi v ch
khi 0 k = hoc
1
2
k = . Tht vy:
*iu kin :
- Khi
1
2
k = , khng nh chng minh cu 1/.
- Ta s tip tc chng minh rng vi k = 1, ba ng trn (O
1
), (O
2
), (O
3
) ln lt i qua
L, tip xc vi OA ti A ; i qua M tip xc vi OB ti B v i qua N tip xc vi OC ti C cng
c ng hai im chung. Tht vy:
- Khi k = 1, cc im L, M, N tng ng trng vi cc im D, E, F.
Theo chng minh cu 1/, ng trn (K, KA) i qua D v tip xc vi OA ti A nn
chnh l ng trn (O
1
) ang c xt. Gi d
1
, d
2
, d
3
l tip tuyn ca ng trn (O) ln lt
ti A, B, C. Gi X, Y, Z theo th t l giao im ca
2 3 3 1 1 2
, ; , ; , d d d d d d .
V O
1
thuc ng thng BC v OA tip xc vi (O
1
) ti A nn O
1
thuc d
1
, t suy ra
O
1
chnh l giao im ca BC v d
1
.
Tng t: O
2
, O
3
ln lt chnh l giao im ca CA v d
2
, AB v d
3
.
Qua cc im O
1
, O
2
, O
3
v cc tip tuyn ti ng trn (O) ln lt l
1 1 2 2 3 3
, , OT O T OT
(trong T
1
, T
2
, T
3
l cc tip im).
Ta c:
1 1 1 2 2 2 3 3 3
, , OT O A O T O B OT O C = = = , tc l T
1
, T
2
, T
3
cng tng ng thuc cc ng
trn (O
1
), (O
2
), (O
3
).
www.VNMATH.com
55

Theo b (4) trn, (xt tam gic XYZ c (O) l ng trn ni tip) cc ng thng
AT
1
, BT
2
, CT
3
tng ng trng vi cc ng thng AX, BY, CZ.
Hn na, XB = XC, YC = YA, ZA = ZA nn:
. . 1
AY BZ CX
AZ BX CY
= AX, BY, CZ ng quy (theo nh l Ceva o trong tam gic XYZ).
Do : AT
1
, BT
2
, CT
3
ng quy. t im chung ca ba ng thng l S, r rng S
nm trong (O). Do T
1
, T
2
, T
3
nm trn (O) nn theo tnh cht phng tch:
1 2 3
2
1 3 / ( ) / ( ) / ( )
. . .
S O S O S O
SAST SB ST SC ST P P P = = = =
.
Tng t cu 1/, ta c:
1 2 3
/( ) / ( ) / ( ) O O O O O O
P P P = = , tc l OS l trc ng phng chung ca ba
ng trn O
1
), (O
2
), (O
3
).
Mt khc, S nm trong c ba ng trn, O nm ngoi c ba ng trn nn ng thng
OS ct c ba ng trn ti hai im, tc l (O
1
), (O
2
), (O
3
) c ng hai im chung.
Vy trong trng hp k = 1, ba ng trn (O
1
), (O
2
), (O
3
) cng c ng hai im chung.
iu kin ca khng nh trn c chng minh.




















S
T
3
T
2
T
1
O
2
O
3
O
1
Z
X
Y
O
C
B
A
www.VNMATH.com
56

*iu kin cn:
Vi mt gi tr 0, 1 k k > , gi
1( ) 2( ) 3( )
, ,
k k k
O O O ln lt l tm ca cc ng trn i qua
L, tip xc vi (O) ti A; i qua M, tip xc vi (O) ti B, i qua N, tip xc vi (O) ti N.
Gi s cc ng trn
( ) ( ) ( )
1( ) 2( ) 3( )
, ,
k k k
O O O ni trn c
ng hai im chung, tc l ba
tm ca chng l
1( ) 2( ) 3( )
, ,
k k k
O O O
thng hng. (1)
Gi d
1
, d
2
, d
3
l tip tuyn
ca ng trn (O) ln lt ti A,
B, C. Gi X, Y, Z theo th t l
giao im ca d
2
, d
3
; d
3
, d
1
; d
1
, d
2
.
Chng minh tng t nh trn,
AX, BY, CZ ng quy. (2)
t O
1
, O
2
, O
3
l giao
im ca BC vi YZ, CA vi ZX,
AB vi XY. D thy rng:
1( ) 2( ) 3( )
, ,
k k k
O O O ln lt thuc cc
on thng
1 2 3
, , AO BO CO
v
1( ) 2( )
1 2
,
k k
AO BO
AL BM
AD BE AO BO
= = ,
3( )
3
k
CO
CN
CF CO
= .
Suy ra:
1( ) 2( ) 3( )
1 2 3
k k k
AO BO CO
k
AO BO CO
= = = . (3)
T (1), (2), (3), p dng b 3, ta c 1 k = hoc
1
2
k = .
Do , nu cc ng trn (O
1
), (O
2
), (O
3
) c ng hai im chung th 1 k = hoc
1
2
k = .
iu kin cn ca khng nh c chng minh.
Vy tt c cc gi tr k cn tm l 1 k = v
1
2
k = .
Bi ton c gii quyt hon ton.


O
1(k)
S
O
2
O
3
O
1
Z
X
Y
O
C
B
A
O
3(k)
O
2(k)
www.VNMATH.com
57

Bi 6. K hiu M l tp hp gm 2008 s nguyn dng u tin. T tt c cc s
thuc M bi ba mu xanh, vng, sao cho mi s c t bi mt mu v mi mu u
c dng t t nht mt s. Xt cc tp hp sau:
3
1
{( , , ) , S x y z M = trong x, y, z c cng mu v( ) 0 (mod 2008)} x y z + + ;
3
1
{( , , ) , S x y z M = trong x, y, z i mt khc mu v( ) 0 (mod 2008)} x y z + + .
Chng minh rng
1 2
2 S S > . (K hiu
3
M l tch cc M M M ) .

*Trc ht ta s chng minh b sau:
Vi n l s nguyn dng, xt tp hp M = {1, 2, 3, , n}. T mu cc phn t ca S bi
mu xanh hoc . Xt cc tp hp sau:
3
1
{( , , ) , S x y z M = trong x, y, z cng mu v( ) 0 (mod )} x y z n + + ;
3
2
{( , , ) , S x y z M = trong x, y, z khc mu v( ) 0 (mod )} x y z n + + .
Gi s rng trong M c a s c t mu v b s c t mu xanh (vi a + b = n) th
2 2
1
| | S a ab b = + ,
2
| | 3 S ab = .

Tht vy:
Ta chn mt s x c t mu , mt s y c t mu xanh, x y . Gi s z l mt s
thuc S m | ( ) n x y z + + , r rng z tn ti v duy nht (z c th trng vi x ho y).
Khng mt tnh tng qut, gi s z c t mu , cng mu vi x.

Xt hai trng hp:
- Nu z khc c x v y. Khi c ba s x, y, z l phn bit. Khi , ta c tt c 6 b ba
thuc tp S
2
cha c x v y l: ( , , ); ( , , ); ( , , ); ( , , ); ( , , ); ( , , ) x y z x z y y x z y z x z x y z y x .
Nu khng tnh n th t ca x v y th ch c 3 b trong cc b trn thuc S
2
. Tht vy: khi xt
x ng trc z trong cc b trn, ta c 3 b ba l: ( , , ); ( , , ); ( , , ) x y z x z y y x z .
Tng t, li xt cc b khng tnh n th t ca z v y (ch rng trm, ta gi s x
v x cng mu), xt x ng sau z trong cc b ny, ta cng c 3 b ba na l:
( , , ); ( , , ); ( , , ) z x y z y x y z x . Do , trong trng hp ny, ta c tt c 3 b thuc S
2
.

- Nu z bng x hoc z bng y. Khng mt tnh tng qut, gi s z = x (trng hp z = y
hon ton tng t, khng quan tm n mu ca chng na). Khi , ta cng c cc b 3 thuc
tp S
2
cha c x v y l: ( , , ); ( , , ); ( , , ) x x y x y x y x x (ch xt cc b khng tnh n th t ca x, y).
Do , trong c hai trng hp, mi b khng tnh n th t (x, y) vi x, y khc mu
nhau cho ta ng 3 phn t trong tp T
2
v mi phn t nh vy xut hin ng mt ln. Suy ra
gi tr ca |S
2
| bng 3 ln s b khng tnh n th t (x, y) nu.
Mt khc: c a s c t mu , b s c t mu xanh nn s b (x, y) ni trn chnh l
ab, t ta c:
2
| | 3 S ab = . Vi x, y cho trc th s z tha mn | ( ) n x y z + + l duy nht.
C x v y c chn ng n ln nn
2 2
1 2
| | ( ) S S n a b = = + , hn na:
1 2
S S = nn
2 2 2 2
1 2 1
| | | | ( ) | | ( ) 3 S S a b S a b ab a ab b + = + = + = + .
B c chng minh.
www.VNMATH.com
58

*Tr li bi ton:
Gi s s cc s c t mu xanh, vng, ln lt l a, b, c th : 2008; , , * a b c a b c + + = .
- Xt tp hp:

3
{( , , ) A x y z M = | x, y, z c t cng mu xanh v 0 (mod 2008)} x y z + + .
Cc tp B, C nh ngha tng t, ng vi cc mu vng v .
- Xt tp hp:

3
{( , , ) AB x y z M = | x, y, z c t bi hai mu xanh, vng v 0 (mod 2008)} x y z + +
Cc tp BC, CA c nh ngha tng t, ng vi cc cp mu vng, v , xanh.
- Xt tp hp :
3
{( , , ) ABC x y z M = |x, y, z c t bi c ba mu xanh, vng, v 0 (mod 2008)} x y z + +
Tip theo, ta s dng b trn nh gi s phn t ca cc tp hp trn:
Gi c l mu i din cho hai mu xanh v vng, khi : s b ba c t cng mu chnh l:
A B C AB v s b ba t khc mu chnh l: ABC BC CA .
Ta c:
2 2
| | | | | | | | ( ) ( ) ,| | | | | | 3 ( ) A B C AB a b c a b c ABC BC CA c a b + + + = + + + + + = + .
Hon ton tng t, ta c:
2 2
| | | | | | | | ( ) ( ) ,| | | | | | 3( ( ) A B C BC b c a b c a ABC CA AB a b c + + + = + + + + + = + .
2 2
| | | | | | | | ( ) ( ) ,| | | | | | 3 ( ) A B C CA c a b c a b ABC AB BC b c a + + + = + + + + + = + .
Theo cch xc nh nh trn th:
1 1
| | | | | | | | S A B C S A B C = = + + ,
2 2
| | | | S ABC S ABC = =
Cng tng v tng ng ca nhm th nht ri nhn vi hai, ta c:
2 2 2
6(| | | | | |) 2(| | | | | |) 3( ) A B C AB BC CA a b c + + + + + = + + .
Cng tng v tng ng ca nhm th hai, ta c:
3| | 2(| | | | | |) 6( ) ABC AB BC CA ab bc ca + + + = + + .
Suy ra:
2 2 2
6(| | | | | |) 3| | 3 ( ) ( ) ( ) 0 A B C ABC a b b c c a ( + + = + +

.
Do :
1 2
2 | | | | 0 S S .
ng thc khng xy ra v khng tn ti a b c = = nguyn dng v 2008 a b c + + = .
Vy bt ng thc trn l thc s, tc l
1 2 1 2
2 | | | | 0 2 | | | | S S S S > > .
y chnh l pcm.









www.VNMATH.com
59

LI GII THI CHN I TUYN QUC GIA
D THI IMO 2009

Bi 1. Cho tam gic nhn ABC ni tip ng trn (O). Gi
1 1 1
, , A B C v
2 2 2
, , A B C
ln lt l cc chn ng cao ca tam gic ABC h t cc nh , , A B C v cc im i
xng vi
1 1 1
, , A B C qua trung im ca cc cnh , , BC CA AB. Gi
3 3 3
, , A B C ln lt l cc
giao im ca ng trn ngoi tip cc tam gic
2 2 2 2 2 2
, , AB C BC A CA B vi (O).
Chng minh rng:
1 3 1 3 1 3
, , A A B B C C ng quy.

Ta s chng minh cc ng thng
1 3 1 3 1 3
, , A A B B C C cng i qua trng tm ca tam gic
ABC. Tht vy:
Gi M l trung trc ca BC, A l im i xng vi A
qua trung trc ca BC. Ta s chng minh rng A trng
vi A
3
hay ng trn (AB
2
C
2
) ct (O) ti A.






Ta c: A, A i xng nhau qua trung trc ca BC
nn: , AB A C AC A B = = . Do A, B v C
1
, C
2

cng i xng vi nhau qua trung im ca AB nn
2 1
BC AC = . Tng t:
2 1
CB AB = . Suy ra:

2 1
2 1
'
'
BC AC AC A B
CB AB AB A C
= = = .
Kt hp vi

3 3
' ' C BA B CA = ( cng chn cung AA)
ta c:

2 2 2 2 2 2
' ' ( . . ) ' ' ' ' C BA B CA c g c BC A CB A AC A AB A = = .
Do , t gic AC
2
B
2
A l t gic ni tip hay A trng vi A
3
. Gi G l giao im ca
trung tuyn AM vi A
1
A
3
. Do AA
3
// A
1
M nn:
3
1
2
AA AG
GM AM
= = G l trng tm ca tam gic
ABC hay ng thng A
1
A
3
i qua trng tm G ca tam gic ABC.
Tng t:
1 3 1 3
, B B C C cng i qua G.
Vy cc ng thng
1 3 1 3 1 3
, , A A B B C C ng quy. Ta c iu phi chng minh.
3
3
3
2
2
2
1
1
1
O
B
A
C
C
C
B
A A
B
A
B
C
1
2
2 1
1
G
A'
B
C
C
B
M A
A
B
C
www.VNMATH.com
60

Bi 2.
Cho a thc
3 2
( ) 1 P x rx qx px = + + + trong , , p q r l cc s thc v 0 r > .
Xt dy s sau:
2
1 2 3
3 2 1
1, ,
. . . , 0
n n n n
a a p a p q
a p a q a r a n
+ + +
= = =


Chng minh rng: nu a thc ( ) P x c mt nghim thc duy nht v khng c
nghim bi th dy s ( )
n
a c v s s m.


* Gi s k l mt nghim (thc hoc phc) ca a thc:
3 2
( ) Q x x px qx r = + + + , do r > 0 nn
3 2
0 0 k k pk qk r + + + = (*)
Theo gi thit, a thc
3 2
( ) 1 P x rx qx px = + + + c ng mt nghim thc nn n cn c thm
hai nghim phc lin hp na, ng thi
1
k
chnh l nghim ca P(x) do:
3 2
2 3
3
1 1 1 1
( ) 1 0
r qk pk k
P r q p
k k k k k
+ + + | | | | | |
= + + + = =
| | |
\ \ \
.

Xt dy s (u
n
) xc nh bi cng thc:

1 3 2 1
( )
n n n n
r
u a p k a a
k
+ + + +
= + + (**)
Mt khc, theo gi thit:
3 2 1
, 0,1, 2,...
n n n n
a pa qa ra n
+ + +
= =
Ta c:
1 2 1 2 1 2 1
2 1 2
( ) .
( . )
n n n n n n n n n
n n n
r kq r
u pa qa ra p k a a ka a ra
k k
kq r r
k a a a
k k
+ + + + + + +
+ +
+
= + + =
+
=

T (*)
2 3
2
( )
kq r
kq r pk k p k
k
+
+ = + = + , do :
1 2 1
( ( ) ) , 0,1, 2,...
n n n n n
r
u k a p k a a ku n
k
+ + +
= + + = =
Trong (**), cho n = -1, ta c:
2 3
2 2
0 2 1 0
( ) ( )
r r pk qk r k
u a p k a a p q p k p k
k k k k
+ +
= + + = + = = =
Suy ra:
2 2
2 1
( ) , 0,1, 2,...
n n
n n n n
r
u k a p k a a k n
k
+ +
+ +
= + + = = (***)
www.VNMATH.com
61

Gi s z l nghim phc ca phng trnh P(x) = 0 v , ln lt l modun v argument
ca z trong : , , 0 > .
Ta c: (cos sin )
i
z e i

= = + v ( ) [ ] P x x nn: ( ) 0 ( ) 0 ( ) 0 P z P z P z = = = do :
(cos sin )
i
z e i

= = cng l nghim ca P(x).


T (***), ta c:
2 2
2 1 2 1
( ) ( ) , ( ) ( )
n n
n n n n n n
r r
a p z a a z a p z a a z
z z
+ +
+ + + +
+ + = + + = .
Theo cng thc Moavre, ta c: (cos sin ) (cos sin )
n n
z i z n i n = + = + nn:
( )
| | | |
| |
( )
| | | |
2
2 2 2
2
2
2
2 1
cos( 2) sin( 2) cos( 2) sin( 2)
2 . sin ( 2) sin ( 2)
2 . sin ( 2) .
2 . sin sin
n
n n n
n
n
n
n n
z z n i n n i n
z z
i n n
i n
i z z




+
+ + +
+
+
+
+ +
= + + + + + =

+ +
= + = =


Tr tng v hai ng thc:
2 2
1
1 1
( ) ( ) ( ) ( )
n n
n n
z z a r a z z
z z
+ +
+
=
| |
2 2
2 2
1 1 2
1
1 2
( )
( ) ( ) ( )
. ( )
sin ( 2)
.
sin
n n
n n
n n n n
n
n n
z z r z z
z z a r a z z a a
z z z z
n
r
a a


+ +
+ +
+ +
+
+

= + =

+
+ =


Do 0 > nn xt n
0
l mt gi tr nguyn dng sao cho:
| | | |
0
0 0
0 0 1
1 2
sin ( 2) sin ( 2)
0 . 0 0
sin sin
n
n n
n n
r
a a


+
+
+ +
< < + < .
V
2
0
r

> nn
0 0
1
,
n n
a a
+
tri du vi nhau. Do , trong hai gi tr ny c mt s m.
Ta thy khi n tin ti v cc, tn ti v s gi tr n
0
sao cho
| |
0
sin ( 2)
0
sin
n

+
< , m ng vi
mi gi tr n
0
nh th ta li tm c mt s hng m ca dy cho, tc l dy (a
n
) c v s s
m. y chnh l iu phi chng minh.

www.VNMATH.com
62

Bi 3.
Cho cc s nguyn dng a, b sao cho , a b v . a b u khng l s chnh phng.
Chng minh rng trong hai phng trnh sau:

2 2
2 2
1
1
ax by
ax by
=
=

c t nht mt phng trnh khng c nghim nguyn dng.


* Trc ht ta s chng minh b sau:
Cho A, B l cc s nguyn dng v A, B, AB u khng l cc s chnh phng.
Khi : nu gi (a, b) l nghim nguyn dng nh nht ca phng trnh Pell
2 2
1 x ABy =
(do AB khng l s chnh phng nn phng trnh Pell ny lun c nghim nguyn dng,
ngha l (a, b) tn ti) v (x
0
, y
0
) l nghim nguyn dng nh nht ca phng trnh
2 2
1 Ax By = th ta lun c h thc lin h sau:
2 2
0 0
0 0
2
a Ax By
b x y
= +


*Chng minh:
Do (x
0
,y
0
) l nghim ca
2 2
1 Ax By = nn
2 2
0 0
1 Ax By = .
t
2 2
0 0 0 0
, 2 u Ax By v x y = + = ; khi , ta c:
( ) ( ) ( )
2 2
2
2 2 2 2 2 2
0 0 0 0 0 0
. 2 1 u ABv Ax By AB x y Ax By = + = = .
Do , (u; v) l mt nghim ca
2 2
1 x ABy = .
M (a, b) l nghim nguyn dng nh nht ca phng trnh Pell
2 2
1 x ABy = nn , u a v b .
Ta s chng minh rng u = a, v = b.
Tht vy, gi s ngc li, u > a, v > b. Ta c:
( )( )
( )( )
( )
2 2
0 0 0 0
0 0 0 0 0 0
1
( )
a b AB a b AB a b AB a ABb
a b AB Ax By Ax By
ax Bby A ay Abx B Ax By
< + = =
+ < +
+ < +

Mt khc:
( )
2
2 2
0 0 0 0 0 0
2 a b AB u v AB Ax By x y AB Ax By + < + = + + = +
Do :
( )
( ) ( )
( ) ( ) ( )( ) ( )
0 0 0 0 0 0
2
2 2
0 0 0 0 0 0 0 0 0 0
( ) ax Bby A ay Abx B a b AB x A y B
x A y B x A y B Ax By x A y B x A y B
= + <
+ = + = +

t
0 0 0 0
, s ax Bby t ay Abx = = , ta c:
0 0
0 0
(1)
(2)


s A t B x A y B
s A t B x A y B

+ < +

< +


www.VNMATH.com
63

Hn na: ( ) ( ) ( )( )
2 2
2 2 2 2 2 2
0 0 0 0 0 0
1.1 1 As Bt A ax Bby B ay Abx a ABb Ax By = = = =
Ta thy s > 0 v:
2 2 2 2 2 2 2 2 2
0 0 0 0 0 0 0 0
0 0 ( 1) s ax Bby ax Bby a x B b y a x Bb Ax > > > > >
2 2 2 2 2 2
0 0
( ) a ABb x Bb x Bb > > , ng do B > 0.
Ta cng thy 0 t bi v nu t = 0 th:
2 2 2 2 2 2 2 2 2 2 2
0 0 0 0 0 0 0 0 0
0 ( 1) ( 1) ay Abx ay Abx a y A b x y ABb Ab By y Ab = = = + = + =
iu ny mu thun do A khng l s chnh phng.
-Nu t > 0 th (s; t) l mt nghim dng ca phng trnh
2 2
1 Ax By = , t :
, s a t b , suy ra:
0 0
s A t B x A y B + > + , iu ny mu thun vi (1).
-Tng t, nu t < 0 th (s; -t) l mt nghim dng ca phng trnh
2 2
1 Ax By = , t :
, s a t b , suy ra:
0 0
s A t B x A y B > + , iu ny mu thun vi (2).
Do , iu gi s l sai, ngha l u = a, v = b. B c chng minh.

* Tr li bi ton:
Gi s ngc li, c hai phng trnh:
2 2
2 2
1 (*)
1 (**)


ax by
ax by
=
=

u c nghim nguyn dng. Gi (m, n) l nghim nguyn dng nh nht ca phng trnh
2 2
1 x aby = ; (x
1
; y
1
) l nghim nguyn dng nh nht ca (*); (x
2
; y
2
) l nghim nguyn
dng nh nht ca (**). Theo b trn, ta c cc h thc sau:

2 2
1 1
1 1
2
m ax by
n x y
= +

v
2 2
2 2
2 2
2
m bx ay
n x y
= +


T (*)
2 2
1 1
1 ax by = + , t (**)
2 2
2 2
1 ay bx = , so snh cc ng thc trn, ta cng c:
( )
2 2 2 2 2 2 2 2
1 1 2 2 1 2 1 2
2 1 2 1 1 ax by bx ay by bx b y x + = + + = = .
Nhng do b l s nguyn dng, khng phi l s chnh phng nn b > 1, ngha l ng thc
trn khng th xy ra. Suy ra iu gi s trn l sai.
Vy trong hai phng trnh (*) v (**) cho c t nht mt phng trnh khng c nghim
nguyn dng. y chnh l iu phi chng minh.




www.VNMATH.com
64

Bi 4.
Tm tt c cc s thc r sao cho bt ng thc sau ng vi mi s , , a b c dng:
3
1

2
a b c
r r r r
b c c a a b
| || || | | |
+ + + +
| | | |
+ + +
\ \ \ \


Ta s xt iu kin cn v tm cc gi tr r tha bi.
* iu kin cn: Xt trng hp 0 a b = > . t 0
c
t
a
= > . Ta c:
3
2
2 3 3
1

2
1 1 1
. .
2 2 2 2
1
a b c
r r r r
b c c a a b
a c c
r r r r r r
c
a c a a
a
| || || | | |
+ + + +
| | | |
+ + +
\ \ \ \
| |
|
| | | | | | | | | |
+ + + + + +
|
| | | | |
+
\ \ \ \ \
|
+
\

2 3 2 3
2
2
2
1 1 2 1 1
. .
1 2 2 1 1 2 2
2 3 1 2 3 1
. . 0
1 2 2 ( 1) 1 4 1 8
t r t
r r r r r r
t t t
t t t
r r
t t t t
(
| | | | | | | | | | | |
+ + + + + + +
(
| | | | | |
+ + +
\ \ \ \ \ \
(

| |
| | | |
+ + + +
| | |
+ + + +
\ \
\

Cho 0 t , t bt ng thc trn, suy ra:
2
5 1
4
4 2 1 0 (*)
5 1
4
r
r r
r


* iu kin :
Ta s chng minh rng vi gi tr r tha mn (*) th bt ng thc cho ng vi mi s dng
a, b, c. Tht vy: t: , , , , , 0
a b c
x y z x y z
b c c a a b
= = = >
+ + +
. Ta c:
2 . . . . .
( ) ( ) ( ) 2 ( )( )( )
1
( )( )( ) ( )( )( )
a b b c c a a b c
xy yz zx xyz
b c c a c a a b a b b c b c c a a b
ab a b bc b c ca c a abc a b b c c a
a b b c c a a b b c c a
+ + + = + + + =
+ + + + + + + + +
+ + + + + + + + +
= =
+ + + + + +

Ta s chng minh cc bt ng thc sau:
(1) 2( )
2
( )( ) ( )( ) ( )( )
( )( ) ( )( ) ( )( ) 2 ( ) 2 ( ) 2 ( )
( )( )( ) ( )( )( )
x y z xy yz zx
a b c ab bc ca
b c c a a b a c b c b a c a c b a b
a a b a c b b c b a c c a c b ab a b bc b c ca c a
a b b c c a a b b c c a
+ + + +
(
+ + + +
(
+ + + + + + + + +

+ + + + + + + + + + + + +

+ + + + + +

www.VNMATH.com
65

3 3 3
3 3 3
2 2 2
( ) ( ) ( ) 3 2 ( ) 2 ( ) 2 ( )
3 ( ) ( ) ( )
( ) ( ) ( ) 0
( )( ) ( )( ) ( )( ) 0
a b c ab a b bc b c ca c a abc ab a b bc b c ca c a
a b c abc ab a b bc b c ca c a
a a bc ab ac b b ca ba bc c c ab ca cb
a a b a c b b a b c c c a c b
+ + + + + + + + + + + + + +
+ + + + + + + +
+ + + + +
+ +

Bt ng thc ny chnh l bt ng thc Schur vi cc s dng a, b, c.
| | | |
3
(2)
4
3
( )( ) ( )( ) ( )( ) 4
3
( ) ( ) ( ) ( )( )( )
4
4 ( ) ( ) ( ) 3 ( ) ( ) ( ) 6
( ) ( ) (
xy yz zx
ab bc ca
a c b c b a c a c b a b
ab a b bc b c ca c a a b b c c a
ab a b bc b c ca c a ab a b bc b c ca c a abc
ab a b bc b c ca c
+ +
+ +
+ + + + + +
+ + + + + + + +
+ + + + + + + + + + +
+ + + + +
2 2 2
) 6
( ) ( ) ( ) 0
a abc
a b c b c a c a b

+ +

Bt ng thc cui ng nn (2) ng vi mi s dng a, b, c.
t
3
4
t xy yz zx t = + + . Bt ng thc cho chnh l:
3
3 2 3 2
2
1
( )( )( ) ( )
2
3 3 1
( ) ( )
2 4 8
3 3 1
( ) ( ) 0
2 4 8
r x r y r z r
r x y z r xy yz zx r xyz r r r
x y z r xy yz zx r xyz
+ + + +
+ + + + + + + + + +
+ + + + + +

Do 2( ) x y z xy yz zx + + + + v
1
2 1 (1 )
2
xy yz zx xyz xyz xy yz zx + + + = = nn chng
minh bt ng thc trn, ta chng minh bt ng thc sau:
2
2
2 2
2 2
2
3 3 1 1
2( ) ( ) (1 ) 0
2 4 2 8
3 3 1 1
(2 ) ( ) (1 ) 0
2 4 2 8
1 3 3 3
(2 )
2 2 4 8
4 2 1 3(4 2 1) 3
. (4 2 1) 0
2 8 4
xy yz zx r xy yz zx r xy yz zx
t r t r t
t r r r r
r r r r
t r r t
(
+ + + + + +
(

+ +
+ +
+ + | |
+
|
\

Bt ng thc cui ng do (*) v (2) nn bt ng thc cho l ng.
Vy iu kin ca r cn tm l
5 1
4
r

hoc
5 1
4
r

.
www.VNMATH.com
66

Bi 5.
Cho ng trn (O) c ng knh AB v M l mt im bt k nm trong (O), M
khng nm trn AB. Gi N l giao im ca phn gic trong gc M ca tam gic AMB vi
ng trn (O). ng phn gic ngoi gc AMB ct cc ng thng NA, NB ln lt ti
P, Q. ng thng MA ct ng trn ng knh NQ ti R, ng thng MB ct ng
trn ng knh NP ti S v R, S khc M.
Chng minh rng: ng trung tuyn ng vi nh N ca tam gic NRS lun i qua
mt im c nh khi M di ng pha trong ng trn.


















* Qua R k ng thng song song vi PQ ct NA ti C, qua S k ng thng song song vi
PQ ct NB ti D. Gi I l trung im ca CD .
Ta s chng minh rng CD // AB.
Tht vy, do N nm trn ng trn ng knh AB nn:

0
90 ANB AN BN = , suy
ra BN l tip tuyn ca ng trn ng knh PN.
Do : ( . ) BMN BNS g g
V PQ l ng phn gic gc ngoi ca AMN nn:

SMP AMP QMR BMQ = = = .
Mt khc:

SMP SNP = (gc ni tip cng chn cung PS ca ng trn ng knh PN),


QMR QNR = ( gc ni tip cng chn cung QR ca ng trn ng knh QN).
Do :

SNP QNR SNP SNR QNR SNR CNR SNB = + = + = .
Xt hai tam gic BNS v RNC c:

CNR SNB = v

RCN MPN NSM NSB = = =
nn: ( . ) BNS RNC g g .
Suy ra cc tam gic ng dng: BMN BNS RNC .
C
D
I
S
R
M
P
Q
N
B
O
A
www.VNMATH.com
67

Tng t, ta cng c: DSN RAN NAM .
* Ta thy, t: . .
NB NS
BNS RNC NB NC NR NS
NR NC
= =
. .
NS ND
DSN RAN NA ND NR NS
NA NR
= = .
Suy ra: . .
NA NC
NA ND NB NC
NB ND
= = AB // CD
Trung im ca AB, trung im ca CD v N l ba im thng hng.
Tc l N, O, I thng hng. (1)
* Hn na:
. MN BN NB NC
BMN RNC RC
NC RC MN
= = .

. DN DS NA ND
DSN NAM DS
MN NA MN
= = .
Kt hp cc iu trn, ta c: RC = DS, m RC // DS (cng song song vi PQ) nn t gic
RCSD l hnh bnh hnh.
Do , hai ng cho CD v RS ca t gic ct nhau ti trung im ca mi ng.
Suy ra I l trung im ca CD cng l trung im ca RS.
Khi : NI chnh l ng trung tuyn ca tam gic NRS. (2)
T (1) v (2), suy ra: trung tuyn NI ca tam gic NRS lun i qua O.
Vy trung tuyn ng vi nh N ca tam gic NRS lun i qua I l im c nh khi M di ng
khp pha trong ng trn (O).
y chnh l iu phi chng minh.

www.VNMATH.com
68

Bi 6.
Mt hi ngh ton hc c tt c 6 4 n + nh ton hc phi hp vi nhau ng 2 1 n +
ln ( ) 1 n . Mi ln hp, h ngi quanh mt ci bn 4 ch v n ci bn 6 ch, cc v tr ngi
chia u khp mi ci bn. Bit rng hai nh ton hc ngi cnh hoc i din nhau
mt cuc hp ny th s khng c ngi cnh hoc i din nhau mt cuc hp khc.
a/ Chng minh rng Ban t chc c th xp c ch ngi nu 1 n = .
b/ Hi rng Ban t chc c th sp xp c ch ngi c hay khng vi mi 1 n > ?

a. Vi 1 n = , ta c bi ton nh sau: mt hi ngh ton hc c 10 nh ton hc, h phi hp
vi nhau ng 3 ln v trong mi ln hp nh th, h phi ngi quanh mt ci bn 4 ch v mt
ci bn 6 ch, cc v tr ngi chia u khp bn; ng thi, hai ngi ngi k nhau hoc i
din nhau trong cuc hp ny th khng c ngi k nhau hoc i din nhau trong mt cuc
hp khc.
nh s th t cho 10 nh ton hc ang xt l (1), (2), (3), (10).
Ta s ch ra mt cch sp xp tha mn bi trong trng hp ny. Ta c cc s sau:
-Bui hp th 1:







-Bui hp th 2:







-Bui hp th 3:








7
8
9
6
10
4
2
1
3
5
3
8
4
6
10
9
5
1
7
2
7
3
9
2
4
10
6
1
8
5
www.VNMATH.com
69

b. Ta s chng minh rng trong trng hp tng qut, Ban t chc lun c th xp ch ngi
cho cc nh ton hc trong cc cuc hp.
Ta chia 6 4 n + nh ton hc thnh 2 2 n + nhm. Mt nhm ch gm 1 ngi lun
ngi mt v tr c nh ti bn 4 ch, t ngi ny l X
0
; 2 1 n + nhm cn li chia ra t
6 3 n + nh ton hc, mi nhm c 3 nh ton hc.
t cc nhm l
1 2 3 2 1
, , ,...,
n
X X X X
+
. Cc nhm ny s ln lt ngi vo cc v tr cn li
ca bn 4 ch cng vi X
0
, mi nhm ngi ng mt ln.
* Vi cc bn 6 ch, ta c cch sp xp nh sau:
bc th , 1 2 1 k k n + , vi hai nhm bt k ,
i j
X X trong : (mod 2 1), i j k n + +
1 , 2 1, i j n i j + th cc nh ton hc thuc hai nhm X
i
, X
j
s ngi vo hp cng nhau
mt bn 6 ch no ; ng thi, nhng nh ton hc thuc cng mt nhm th ngi cc v
tr to thnh mt tam gic u trn cc bn 6 ch, ngha l h s khng ri vo trng hp ngi
i din nhau hoc ngi cnh nhau.












* Vi bn 4 ch, ta c cch sp xp nh sau:
- Nu k l s chn th bc ny, ch c mt nhm c ch s
2
k
l khng c ngi
chung bn 6 ch vi nhm no, nh th nhm ny s ngi vo bn 4 ch cng vi X
0
.
- Nu k l s l, bc ny; tng t trn, ch c mt nhm c ch s l
2 1
2
k n + +

khng c chung bn 6 ch vi nhm no, nhm ny s ngi vo bn 4 ch cng vi X
0
.
D dng thy rng cch sp xp nh th tha mn mi yu cu ca bi ton.
Vy Ban t chc c th sp xp c ch ngi mi 1 n > .
j
i
X
X
www.VNMATH.com
70

LI GII THI CHN I TUYN QUC GIA
D THI IMO 2010
Bi 1.
Cho tam gic ABC khng vung ti A c ng trung tuyn AM. Gi D l mt im di
ng trn ng thng AM. Gi ( ) ( )
1 2
, O O l cc ng trn i qua D, tip xc vi BC ln
lt ti B v C. Gi P, Q ln lt l giao im ca ng thng AB vi ng trn (O
1
),
ng thng AC vi ng trn (O
2
). Chng minh rng:
1. Tip tuyn ti P ca
1
( ) O v tip tuyn ti Q ca
2
( ) O phi ct nhau ti mt im.
Gi giao im l S.
2. im S lun di chuyn trn mt ng thng c nh khi D di ng trn AM.
1. V M l trung im ca BC nn
2 2
MA MB = , suy ra M c cng phng tch n hai
ng trn ( ) ( )
1 2
, O O hay M thuc trc ng phng ca .
Hn na, hai ng trn ct nhau ti D nn D cng nm trn trc ng phng ca chng.
T , suy ra DM chnh l trc ng phng ca ( ) ( )
1 2
, O O , m A thuc ng thng
DM nn A c cng phng tch n hai ng trn ( ) ( )
1 2
, O O .
Suy ra: . . AP AB AQ AC = hay t gic BPQC ni tip.
T h thc trn ta cng thy rng nu P trng vi A th Q cng trng vi A, nu P thuc on
AB th Q cng thuc on AC v ngc li.
x
S
Q
P
O
2
O
1
M
A
B
C
D
www.VNMATH.com
71

Khng mt tnh tng qut, gi s gc

ABC nhn, gi Px l tia tip tuyn ca ng trn


(O
1
) sao cho gc

xPB nhn.
Theo tnh cht tip tuyn, ta c:

xPB PBC = , m

PBC AQP = nn

xPB AQB = .
Px cng l tip tuyn ca ng trn (APQ).
Do : (O
1
) tip xc vi ng trn (APQ).
Hon ton tng t: ta cng c (O
2
) cng tip xc vi ng trn (APQ).
Suy ra: tip tuyn ti P ca
1
( ) O v tip tuyn ti Q ca
2
( ) O cng chnh l hai tip tuyn ca
(APQ) ti cc im P, Q.
Hn na, theo gi thit:

0
90 PAQ nn hai tip tuyn khng song song v do
chng phi ct nhau (pcm).
2. Theo chng minh trn, ta c: S thuc tip tuyn ca (O
1
) v (O
2
), SP SQ = nn S c
cng phng tch n hai ng trn ( ) ( )
1 2
, O O nn S thuc trc ng phng ca hai ng
trn ny, ngha l S nm trn AM.
Vy khi D thay i trn AM th S cng di chuyn trn AM l ng thng c nh.
Ta c pcm.


www.VNMATH.com
72

Bi 2.
Vi mi s n nguyn dng, xt tp hp sau :
{ }
11( ) 10( ) | 1 , 10
k h
n
T k h n n k h = + + + .
Tm tt c gi tr ca n sao cho khng tn ti , ;
n
a b T a b sao cho ( ) a b chia ht cho 110.
t ( , , ) 11( ) 10( ), , ,
k h
f k h n k h n n k h n = + + + .
Ta c: ( , , ) ( , , ) f k h n f h k n = nn khng mt tnh tng qut, ta gi s h k .
Nu (mod11) m n th :
( , , ) ( , , ) 11( ) 10( ) 11( ) 10( )
10 ( ) ( ) 110 ( , , ) ( , , ) (mod110)
k h k h
k k h h
f k h m f k h n k h m m k h n n
m n m n f k h m f k h n
( ( = + + + + + + =

( = +


T , ta ch cn xt cc gi tr n tha 1 11 n .
Xt hiu:
5
(6, 6, ) (1,1, ) 110 20 .( 1) f n f n n n = + . Nu
5
20 .( 1) n n chia ht cho 110 th
gi tr n tng ng s khng tha. T , ta loi i cc gi tr 1, 3, 4, 5, 9,11 n = .
Ta cng c
8 2 6 4 2 2 2 2
(8, 2, ) (6, 4, ) 10( ) 10 ( 1) ( 1) f n f n n n n n n n n = + = + , vi 10 n =
th
2 2 2 2
10 ( 1) ( 1) 110 n n n + nn gi tr ny cng khng tha.
Ta s chng minh rng cc gi tr 2, 6, 7, 8 n = tha mn. Tht vy:
Tnh ton trc tip, ta thy rng vi 2, 6, 7, 8 n = th (mod11)
k h
n n vi (mod11) k h .
Gi s ngc li, vi cc gi tr n nu trn, tn ti hai b ( , ) ( ', ') k h k h (gi s ' k k > ) sao cho:
( , , ) ( ', ', ) f k h n f k h n . Khi :
' '
11( ' ') 10( ) 0 (mod110)
k h k h
k h k h n n n n + + + .
Suy ra:
' ' ' ' '
' ' (mod10), (mod11) ( 1) ( 1) (mod11)
k h k h k k k h h h
k k h h n n n n n n n n

+ + .
Do 11 l s nguyn t nn theo nh l Fermat nh th:
' '
(mod11)
k k h h
n n

.
D thy:
' '
1 1 0 (mod11)
k k h h
n n

nn t ng thc trn, suy ra:
'
(mod11) '
k h
n n k h = .
Do : ' k h = hay ( , ) ( ', ') k h k h , mu thun.
Suy ra cc gi tr 2, 6, 7, 8 n = u tha mn yu cu ca bi.
Vy tt c cc gi tr n cn tm l: 2, 6, 7, 8 (mod11) n .
www.VNMATH.com
73

Bi 3.
Gi mt hnh ch nht c kch thc 1 2 l hnh ch nht n v mt hnh ch nht
c kch thc 2 3 , b i 2 gc cho nhau (tc c c 4 vung con) l hnh ch nht kp.
Ngi ta ghp kht cc hnh ch nht n v hnh ch nht kp ny li vi nhau c mt
bng hnh ch nht c kch thc l2008 2010 .
Tm s nh nht cc hnh ch nht n c th dng ghp.

Ta s chng minh rng s hnh ch nht n nh nht tha mn bi l 1006. Tht vy:
*iu kin cn:
Ta xt mt cch ph hnh ch nht 2008 2010 tha mn bi (ch rng 2008 ch s
hng v 2010 ch s ct). Gi , , , x y z t l s cc hnh ch nht 1 2, 2 1, 2 3, 3 2 trong cch
ph ( y thc ra cc hnh ch nht 1 2, 2 1 u l cc hnh ch nht n ca bi, ch
phn bit cch ph dc hay ngang, trong 1 2 c t ngang, 2 1 c t dc; tng t
vi cch phn bit 2 3, 3 2 ).
T mu trng cho cc hng l, t mu en cho cc hng chn. tt c cc ca hng th
,1 2010 i i , ta nh s tng ng cc s t nhin i.
Ta s chng minh cc nhn xt sau:
- Nhn xt 1: ta lun c ng thc: 2( ) 4( ) 2008.2010 x y z t + + + = .
Mi hnh ch nht 1 2 hoc 2 1 c cha hai vung, mi hnh ch nht 2 3 hoc
3 2 c cha bn vung. Tng cc vung ny bng s vung ca c hnh ch nht ln nn
2( ) 4( ) 2008.2010 x y z t + + + = .
- Nhn xt 2: Gi tr , x y l chn.
Ta thy trn ton bng, cc hnh ch nht 2 3, 3 2 u c s trng bng s en; cc
hnh ch nht 2 1 c t dc nn cng c s trng bng s en. Suy ra, s hnh ch nht
1 2 cc hng c t mu trng bng s hnh ch nht cc hng c t en. Hn na, tng
s cc hng l 2010 l chn nn gi tr x phi l chn. T nhn xt 1, ta thy y cng phi chn.
Tr li bi ton, ta xt tng ng i t tp hp cc hnh ch nht ang xt n cc s
nguyn l hiu gia tng cc s vung c t en vi tng cc s vung c t trng
ghi trn n.
D dng thy rng: (3 2) 0; (2 3) 2; (2 1) 1 = = = .
T suy ra: (3 2) 0; (2 3) 2 ; (2 1) z y =

.
(k hiu (3 2)

l tng tnh trn tt c cc hnh ch nht 3 2 c dng, nh ngha tng


t vi cc hnh ch nht khc).
Ta cng thy rng, tng cc s ghi trn x hnh ch nht 1 2 l mt s chn thuc | | 2; 2.2008 ,
m x l s chn (nhn xt 2), ta c nh gi sau: ( ) (1 2) 2.2008 2
2
x

.
www.VNMATH.com
74

Ta c:
| |
1004 1004
1 1
(2008 2010) 2010. 2 (2 1) 2010. 2010.1004
i i
i i i
= =
= = =

.
Mt khc: (2008 2010) (2 1) (1 2) (3 2) (2 3) = + + +

.
T cc iu trn, suy ra: 2010.1004 .(2.2008 2) 2 2010.1004 2007 2
2
x
y z x y z + + + + .
Tip theo, ta xt hnh ch nht 2010 2008 (tng t nh trn nhng c 2010 hng v
2008 ct), bt u li cc lp lun v s cc hnh ch nht 1 2, 2 1, 2 3, 3 2 c dng.
Ta xy dng c bt ng thc sau: 2008.1005 2009 2 y x t + + .
Cng hai bt ng thc ny li, ta c:
2008.1005 2010.1004 (2009 2 ) (2007 2 ) 2008 2010 2( ) y x t x y z x y z t + + + + + + = + + + .
Hn na, theo nhn xt 1 th: 2010.1004 ( ) 2( ) x y z t = + + + .
T ta c: 2008.1005 2007 2009 2009( ) x y x y + + .
Suy ra:
2008
1005. 1004
2009
x y + > , m x y + l s chn nn 1006 x y + .
Do , tng cc hnh ch nht n cn dng t nht l 1006. iu kin cn c chng minh.

*iu kin :
Ta s ch ra mt cch ghp hnh ch nht dng ng 1006 hnh ch nht n.
Khi 4 Khi 3 Khi 2
Khi 1

www.VNMATH.com
75

Hnh trn m t cch ghp mt hnh ch nht 10 16 , trong : cc hnh ch nht khuyt c t
bng 5 mu khc nhau (, hng, xanh lam, xanh l cy, xanh m) d dng phn bit; trn
hnh cc khi c t mu xanh l m l cc hnh ch nht n chc chn phi dng, cc khi
mu vng th ty trng hp, c th l hnh ch nht n m cng c th l hnh ch nht khuyt.

* Hnh ch nht 2010 2008 c th c to thnh t hnh trn bng quy tc sau:
- Thm cc dng bng cch chn vo gia mi khi trn cc hnh c dng:

Mi ln ghp nh th th ta c thm c hai hng mi, do 2010 chia ht cho 2 nn khi thc hin
vic ny lin tip mt cch thch hp th khi ny s tng v chiu di, to thnh cc khi mi c
kch thc 2010 4 v mi khi nh vy, ta ch dng ng 2 hnh ch nht mu xanh l m.
- Thm ct bng cch lp li cc khi 1, 2 , 3, 4 trn hnh (ch tnh tun hon gia cc
khi: (1) tng ng vi (3), (2) tng ng vi (4)).
Nh th th ta cn phi c tt c 502 khi dnh cho 2008 ct. ng thi, khi u tin v khi
cui cng, ta cn dng thm mt hnh ch nht n mu vng, cc khi gia th dng cc hnh
ch nht khuyt mu vng. Tc l: hai khi u tin v cui cng, ta cn dng 3 hnh ch nht
n, cc khi gia ch cn dng 2 hnh ch nht n thi.
Khi , tng s hnh ch nht n cn dng l: 500.2 2.3 1006 + = .
Xoay hnh ch nht 2010 2008 li, ta c hnh ch nht 2008 2010 cn phi ghp, hnh ch
nht c ng 1006 hnh ch nht n tha mn bi.
Do , iu kin c chng minh.

Vy gi tr nh nht cc hnh ch nht n cn dng l 1006.
Bi ton c gii quyt hon ton.
www.VNMATH.com
76

Bi 4.
Cho , , a b c l cc s thc dng tha mn iu kin:
1 1 1
16( ) a b c
a b c
+ + + + .
Chng minh rng:
3 3 3
1 1 1 8
9 ( 2( )) ( 2( )) ( 2( )) a b a c b c b a c a c b
+ +
+ + + + + + + + +
.
Hi ng thc xy ra khi no?

Theo bt ng thc Cauchy, ta c:
2
3 3
( )( )
2( ) ( ) 3 ( ). 3
2 2 2 2
a c a c a c a b a c
a b a c a b a b
| |
+ + + + +
+ + + = + + + + =
|
|
\

( )
3
1 2
27( )( )
2( )
a b a c
a b a c

+ +
+ + +
.
Tng t vi hai biu thc cn li. Do :
( )
3
1 2 4( )
27( )( ) 27( )( )( )
2( )
cyc sym
a b c
a b a c a b b c c a
a b a c
+ +
=
+ + + + +
+ + +

.
Hn na, ta thy vi mi a, b, c dng:

2
9( )( )( ) 8( )( ) ( ) 0
8
( )( )( ) ( )( )
9
sym
a b b c c a a b c ab bc ca a b c
a b b c c a a b c ab bc ca
+ + + + + + + =
+ + + + + + +


Do :
( )
3
1 1
6( )
2( )
cyc
ab bc ca
a b a c

+ +
+ + +

. (1).
Mt khc, ta cng c:
2
( ) 3 ( ) ab ca ca abc a b c + + + + nn theo gi thit:
1 1 1 3( ) 3
16( )
16
ab bc ca a b c
a b c ab bc ca
a b c abc ab bc ca
+ + + +
+ + + + = + +
+ +
. (2)
T (1) v (2), suy ra:
3 3 3
1 1 1 8
9 ( 2( )) ( 2( )) ( 2( )) a b a c b c b a c a c b
+ +
+ + + + + + + + +
.
Ta c pcm.
ng thc xy ra khi du bng tt c cc bt ng thc trn xy ra hay:

, , 0
1
4
1 1 1
16( )
a b c
a b c a b c
a b c
a b c

>

= = = = =

+ + = + +

.
www.VNMATH.com
77

Bi 5.
Trong mt hi ngh c n nc tham gia, mi nc c k i din ( 1 n k > > ). Ngi ta
chia n.k ngi ny thnh n nhm, mi nhm c k ngi sao cho khng c hai ngi no
cng nhm n t cng mt nc.
Chng minh rng c th chn ra mt nhm gm n ngi sao cho h thuc cc nhm khc
nhau v n t cc nc khc nhau.

Ta gi mt nc X v mt nhm Y no c lin h vi nhau nu trong nhm Y c
ngi ca nc X. Khi , mt nc X bt k c k ngi i din nn c lin h vi k nc v
mt nhm Y bt k c cha k ngi i din khc nhau ca cc nc khc nhau nn c lin h
vi ng k nc. Do , mt tp hp bt k m nc no trong n nc cho s c lin h vi
t nht
. mk
m
k
= nhm khc nhau.
Gi , 1,
i
a i n = l n nhm cho v ,1,
i
X n l tp hp cc nhm c lin h vi nc th i.
Theo iu va chng minh trn, ta thy vi mi:
1 2 3
, , ,..., {1, 2, 3,..., }
k
i i i i n , 1 k n th:
1
j
k
i
j
X k
=

. (*)
Ta s chng minh rng tn ti
1 2 3 1 2 3
( , , ,..., ) ...
n n
a a a a X X X X v ,
i j
a a i j .
Tht vy, ta bt u b i phn t thuc mi tp X
i
sao cho (*) vn c tha mn. Cui
cng thu c cc tp hp mi
1 2 3
' , ' , ' ,..., '
n
X X X X (vi ' )
i i
X X vn tha iu kin (*) c s
phn t nh nht m nu b i thm bt c phn t thuc tp hp ' ,1,
i
X n th iu kin (*) s
khng cn c tha.
Ta chng minh rng ' 1, 1,
i
X i n = = .
Tht vy, khng mt tnh tng qut gi s
1
' X c cha phn t khc nhau l , . Do nu b
thm mt trong hai phn t hoc th iu kin (*) khng cn tha mn nn s c hai tp ch
s P, Q sao cho:
Vi
1
( ' \ { }) '
i
i P
M X X

,
1
( ' \ { }) '
i
i Q
N X X

khng tha mn iu kin (*), tc l:


1, 1 , M P N Q M P N Q < + < + .
Ta c:

1 1 1
(( ' \ { }) ( ' \ { })) ( ' ' ) ' '
i i i
i P i Q i P Q
M N X X X X X X

= =


'
i
i P Q
X M N


www.VNMATH.com
78

T hai iu ny suy ra: 1 M N P Q + , M N P Q
Theo nguyn l b tr, ta c:
1 1 P Q M N M N M N P Q P Q P Q + + = + + + = + +
iu v l ny dn n khng nh ' 1, 1,
i
X i n = = .
R rng cc tp hp ny khng giao nhau v nu tn ti
, ( ' ' ) ' ' 1 2
i j i j
i j X X X X = < , mu thun vi iu kin (*).
Gi s mi phn t ca '
i
X chnh l '
i
a th tp hp sau:
1 2 3
( ' , ' , ' ,..., ' )
n
a a a a tha mn
' '
i i i
a X X v ,
i j
a a i j .
Vy ta ch ra rng tn ti n nhm khc nhau tng ng lin h vi n nc khc nhau,
mi nhm lin h vi ng mt nc v mi nc lin h ng mt nhm nn n ngi i
din m mi nc lin h vi mi nhm tng ng r rng tha mn bi.
Ta c pcm.

www.VNMATH.com
79

Bi 6.
Gi
n
S l tng bnh phng cc h s trong khai trin ca nh thc (1 )
n
x + , trong
n l s nguyn dng; x l s thc bt k.
Chng minh rng:
2
1
n
S + khng chia ht cho 3 vi mi n.

Ta s chng minh b sau (nh l Lucas):
Cho m, n l hai s t nhin v p l mt s nguyn t. Gi s:

1 2
1 2 1 0
. . ... .
k k
k k
m m p m p m p m p m

= + + + + +

1 2
1 2 1 0
. . ... .
k k
k k
n n p n p n p n p n

= + + + + +
Khi :
0
(mod )
i
i
k
m m
n n
i
C C p
=

(quy c rng 0,
a
b
C a b = > ).
*Chng minh:
Khng mt tnh tng qut, gi s m n > (nu m n = th b hin nhin ng).
Trc ht, ta thy rng: ( , ) 1, 1, 1 p i i p = = nn
! ( 1)!
.
!( )! !( )!
k
p
p p
C p p
k p k k p k

= =

, tc l:
0 (mod ), 1, 1
k
p
C p k p = .
Ta c:
1
1
( 1) 1 . 1(mod )
p
p p i p i p
p
i
x x C x x p

=
+ = + + +

. (*)
Ta s chng minh nhn xt:
*
( 1) 1(mod ),
j j
p p
x x p j + + bng quy np. Tht vy:
- Vi 1 j = , nhn xt ng theo (*).
- Gi s nhn xt ny ng vi 1 j h = . Ta s chng minh rng n cng ng vi 1 j h = + .
Ta c: ( 1) 1 (mod )
h h
p p
x x p + + .
Suy ra:
( ) ( )
1 1
( 1) 1 (mod ) ( 1) 1 (mod )
h h h h
h h
p p p p
x x p x x p
+ +
+ + + + .
Do nhn xt ng vi 1 j h = + . Theo nguyn l quy np, nhn xt c chng minh.
Ta xt khai trin sau:
www.VNMATH.com
80

0
.
.
0 0 0
(1 ) (1 ) (1 ) (mod )
k
i
i
i
i i
i i
i
m k k m p
m m p j j p
m
j i i
x x x C x p
=
= = =

+ = + +

.
H s ca
n
x v (1 )
m
x + l
n
m
C ; do biu din
1 2
1 2 1 0
. . ... .
k k
k k
n n p n p n p n p n

= + + + + + l duy
nht nn h s ca
n
x v
.
0 0
i
i
i
m k
j j p
m
j i
C x
= =

l
0
i
i
k
m
n
i
C
=

.
T ta c:
0
(mod )
i
i
k
m m
n n
i
C C p
=

.B c chng minh.
*Tr li bi ton: Ta c:
2
2 2
2
1 0 0
(1 ) (1 ) .( 1) . . . .
nn n n
n n n i n i i n i n i i
n n n
i i i
x x n C x C x C x

= = =
| | | |
+ = + + =
| |
\ \

.
ng nht h s ca
2n
x hai v, ta c:
2
2
0 0
. ( )
n n
n i n i i
n n n n
i i
C C C C

= =
= =

.
Do , vi mi n t nhin th
2
n
n n
S C = .
Nh th ta cn chng minh rng:
2
4
1
n
n
C + khng chia ht cho 3 vi mi n.
Gi s:
0
2 .3 , , 1,
k
i
i i
i
n a a i k
=
= =

. Xt hai trng hp:


- Nu {0;1}, 1,
i
a i k = th 2 {0; 2}, 1,
i
a i k = v tng {0;1}, 1,
i
a i k = l s chn, t
0
2
0
2 , 2 2 4 1(mod3)
k
i
i
k a
t t
i
i
a t t
=
=

= = =

; ta cng c:
0
4 2 .3 , , 1,
k
i
i i
i
n a a i k
=
= =

.
Theo b trn th
0
2
4 2
0 0
1 1 2 1 2 1 2(mod3)
k
i
i i i
i
k k a
a a n
n a
i i
C C
=
= =

+ + + +

.
- Nu tn ti mt gi tr 2
j
a = ; khng mt tnh tng qut, gi s y l s nh nht trong
tp hp , 0,
i
a i k = . Khi : h s tng ng ti v tr j khai trin theo ly tha 3 ca 4n l 1.
M
2
1
0 C = nn
2 2
4 2 4
0
0 (mod3) 1 1 (mod3)
i
i
k
a n n
n a n
i
C C C
=
+

.
Vy trong mi trng hp, ta u c
2
1
n
S + khng chia ht cho 3. y chnh l pcm.

www.VNMATH.com

You might also like